SlideShare une entreprise Scribd logo
1  sur  546
Télécharger pour lire hors ligne
Nursing Board Practice Test Compilation

                                                                                           FOUNDATION OF PROFESSIONAL NURSING PRACTICE 188

Contents                                                                                      ANSWER KEY - FOUNDATION OF PROFESSIONAL
NURSING PRACTICE I: FOUNDATION OF NURSING                                                     NURSING PRACTICE.................................................. 199
PRACTICE .......................................................................... 4      COMMUNITY HEALTH NURSING AND CARE OF THE
NURSING PRACTICE II ..................................................... 15               MOTHER AND CHILD .................................................... 200

NURSING PRACTICE III .................................................... 26                  ANSWER KEY: COMMUNITY HEALTH NURSING AND
                                                                                              CARE OF THE MOTHER AND CHILD .......................... 211
NURSING PRACTICE IV.................................................... 36
                                                                                           Comprehensive Exam 1................................................ 213
NURSING PRACTICE V..................................................... 46
                                                                                           CARE OF CLIENTS WITH PHYSIOLOGIC AND
TEST I - Foundation of Professional Nursing Practice .... 56
                                                                                           PSYCHOSOCIAL ALTERATIONS...................................... 222
   Answers and Rationale – Foundation of Professional
                                                                                              ANSWER KEY: CARE OF CLIENTS WITH PHYSIOLOGIC
   Nursing Practice ......................................................... 66
                                                                                              AND PSYCHOSOCIAL ALTERATIONS ......................... 234
TEST II - Community Health Nursing and Care of the
                                                                                           Nursing Practice Test V ................................................ 235
Mother and Child ........................................................... 74
                                                                                           Nursing Practice Test V ................................................ 245
   Answers and Rationale – Community Health Nursing
   and Care of the Mother and Child ............................. 84                       TEST I - Foundation of Professional Nursing Practice .. 255

TEST III - Care of Clients with Physiologic and                                               Answers and Rationale – Foundation of Professional
Psychosocial Alterations ................................................ 91                  Nursing Practice ....................................................... 265

   Answers and Rationale – Care of Clients with                                            TEST II - Community Health Nursing and Care of the
   Physiologic and Psychosocial Alterations ................ 102                           Mother and Child ......................................................... 273

TEST IV - Care of Clients with Physiologic and                                                Answers and Rationale – Community Health Nursing
Psychosocial Alterations .............................................. 111                   and Care of the Mother and Child ........................... 283

   Answers and Rationale – Care of Clients with                                            TEST III - Care of Clients with Physiologic and
   Physiologic and Psychosocial Alterations ................ 122                           Psychosocial Alterations .............................................. 290

TEST V - Care of Clients with Physiologic and Psychosocial                                    Answers and Rationale – Care of Clients with
Alterations.................................................................... 133           Physiologic and Psychosocial Alterations ................ 301

   Answers and Rationale – Care of Clients with                                            TEST IV - Care of Clients with Physiologic and
   Physiologic and Psychosocial Alterations ................ 144                           Psychosocial Alterations .............................................. 310

PART III PRACTICE TEST I FOUNDATION OF NURSING . 153                                          Answers and Rationale – Care of Clients with
                                                                                              Physiologic and Psychosocial Alterations ................ 321
   ANSWERS AND RATIONALE – FOUNDATION OF
   NURSING .................................................................. 158          TEST V - Care of Clients with Physiologic and Psychosocial
                                                                                           Alterations.................................................................... 332
PRACTICE TEST II Maternal and Child Health ............... 162
                                                                                              Answers and Rationale – Care of Clients with
   ANSWERS AND RATIONALE – MATERNAL AND CHILD
                                                                                              Physiologic and Psychosocial Alterations ................ 343
   HEALTH..................................................................... 167
                                                                                           PART III ......................................................................... 352
MEDICAL SURGICAL NURSING ..................................... 173
                                                                                           PRACTICE TEST I FOUNDATION OF NURSING .............. 352
   ANSWERS AND RATIONALE – MEDICAL SURGICAL
   NURSING .................................................................. 178             ANSWERS AND RATIONALE – FOUNDATION OF
                                                                                              NURSING .................................................................. 357
PSYCHIATRIC NURSING ................................................ 180
                                                                                           PRACTICE TEST II Maternal and Child Health ............... 361
   ANSWERS AND RATIONALE – PSYCHIATRIC NURSING
   ................................................................................. 185
ANSWERS AND RATIONALE – MATERNAL AND CHILD                                              MEDICAL SURGICAL NURSING Part 1 ........................... 475
    HEALTH..................................................................... 366            ANSWERS and RATIONALES for MEDICAL SURGICAL
MEDICAL SURGICAL NURSING ..................................... 372                             NURSING Part 1 ........................................................ 479
    ANSWERS AND RATIONALE – MEDICAL SURGICAL                                                MEDICAL SURGICAL NURSING Part 2 ........................... 481
    NURSING .................................................................. 377             MEDICAL SURGICAL NURSING Part 2 ....................... 485
PSYCHIATRIC NURSING ................................................ 379                       ANSWERS and RATIONALES for MEDICAL SURGICAL
    ANSWERS AND RATIONALE – PSYCHIATRIC NURSING                                                NURSING Part 2 ........................................................ 489
    ................................................................................. 384   MEDICAL SURGICAL NURSING Part 3 ........................... 491
FUNDAMENTALS OF NURSING PART 1 ........................ 387                                    ANSWERS and RATIONALES for MEDICAL SURGICAL
FUNDAMENTALS OF NURSING PART 2 ........................ 392                                    NURSING Part 3 ........................................................ 495
    ANSWERS and RATIONALES for FUNDAMENTALS OF                                              PSYCHIATRIC NURSING Part 1 ...................................... 497
    NURSING PART 2 ...................................................... 397                  ANSWERS and RATIONALES for PSYCHIATRIC NURSING
FUNDAMENTALS OF NURSING PART 3 ........................ 401                                    Part 1 ........................................................................ 502
    ANSWERS and RATIONALES for FUNDAMENTALS OF                                              PSYCHIATRIC NURSING Part 2 ...................................... 504
    NURSING PART 3 ...................................................... 405                  ANSWERS and RATIONALES for PSYCHIATRIC NURSING
MATERNITY NURSING Part 1 ........................................ 409                          Part 2 ........................................................................ 509
    ANSWERS and RATIONALES for MATERNITY NURSING                                            PSYCHIATRIC NURSING Part 3 ...................................... 512
    Part 1 ........................................................................ 418        ANSWERS and RATIONALES for PSYCHIATRIC NURSING
MATERNITY NURSING Part 2 ........................................ 428                          Part 3 ........................................................................ 516
    Answer for maternity part 2 .................................... 433                    PROFESSIONAL ADJUSTMENT ...................................... 519
PEDIATRIC NURSING .................................................... 434                  LEADERSHIP and MANAGEMENT ................................. 522
    ANSWERS and RATIONALES for PEDIATRIC NURSING                                            NURSING RESEARCH Part 1 .......................................... 532
    ................................................................................. 439   NURSING RESEARCH Part 2 .......................................... 542
COMMUNITY HEALTH NURSING Part 1........................ 444                                    Nursing Research Suggested Answer Key ................ 546
COMMUNITY HEALTH NURSING Part 2........................ 454




2
3
5.    Benner’s “Proficient” nurse level is different
                                                                    from the other levels in nursing expertise in the
NURSING PRACTICE I: FOUNDATION OF NURSING                           context of having:
PRACTICE                                                                a. the ability to organize and plan activities
                                                                        b. having attained an advanced level of
SITUATION: Nursing is a profession. The nurse should                        education
have a background on the theories and foundation of                     c. a holistic understanding and perception
nursing as it influenced what is nursing today.                             of the client
                                                                        d. intuitive and analytic ability in new
    1.   Nursing is the protection, promotion and                           situations
         optimization of health and abilities, prevention
         of illness and injury, alleviation of suffering    SITUATION: The nurse has been asked to administer an
         through the diagnosis and treatment of human       injection via Z TRACK technique. Questions 6 to 10 refer
         response and advocacy in the care of the           to this.
         individuals, families, communities and the
         population. This is the most accepted definition     6.    The nurse prepares an IM injection for an adult
         of nursing as defined by the:                              client using the Z track technique. 4 ml of
              a. PNA                                                medication is to be administered to the client.
              b. ANA                                                Which of the following site will you choose?
              c. Nightingale                                            a. Deltoid
              d. Henderson                                              b. Rectus femoris
                                                                        c. Ventrogluteal
    2.   Advancement in Nursing leads to the                            d. Vastus lateralis
         development of the Expanded Career Roles.
         Which of the following is NOT an expanded            7.    In infants 1 year old and below, which of the
         career role for nurses?                                    following is the site of choice for intramuscular
             a. Nurse practitioner                                  Injection?
             b. Nurse Researcher                                         a. Deltoid
             c. Clinical nurse specialist                                b. Rectus femoris
             d. Nurse anaesthesiologist                                  c. Ventrogluteal
                                                                         d. Vastus lateralis
    3.   The Board of Nursing regulated the Nursing
         profession in the Philippines and is responsible     8.    In order to decrease discomfort in Z track
         for the maintenance of the quality of nursing in           administration, which of the following is
         the country. Powers and duties of the board of             applicable?
         nursing are the following, EXCEPT:                             a. Pierce the skin quickly and smoothly at
              a. Issue, suspend, revoke certificates of                     a 90 degree angle
                 registration                                           b. Inject the medication steadily at around
              b. Issue subpoena duces tecum, ad                             10 minutes per millilitre
                 testificandum                                          c. Pull back the plunger and aspirate for 1
              c. Open and close colleges of nursing                         minute to make sure that the needle did
              d. Supervise and regulate the practice of                     not hit a blood vessel
                 nursing                                                d. Pierce the skin slowly and carefully at a
                                                                            90 degree angle
    4.   A nursing student or a beginning staff nurse who
         has not yet experienced enough real situations       9.    After injection using the Z track technique, the
         to make judgments about them is in what stage              nurse should know that she needs to wait for a
         of Nursing Expertise?                                      few seconds before withdrawing the needle and
             a. Novice                                              this is to allow the medication to disperse into
             b. Newbie                                              the muscle tissue, thus decreasing the client’s
             c. Advanced Beginner                                   discomfort. How many seconds should the nurse
             d. Competent                                           wait before withdrawing the needle?
                                                                         a. 2 seconds

4
5

           b. 5 seconds                                           that the patient smokes and drinks coffee. When
           c. 10 seconds                                          taking the blood pressure of a client who
           d. 15 seconds                                          recently smoked or drank coffee, how long
                                                                  should the nurse wait before taking the client’s
10.    The rationale in using the Z track technique in an         blood pressure for accurate reading?
       intramuscular injection is:                                    a. 15 minutes
           a. It decreases the leakage of discolouring                b. 30 minutes
               and irritating medication into the                     c. 1 hour
               subcutaneous tissues                                   d. 5 minutes
           b. It will allow a faster absorption of the
               medication                                   15.   While the client has pulse oximeter on his
           c. The Z track technique prevent irritation            fingertip, you notice that the sunlight is shining
               of the muscle                                      on the area where the oximeter is. Your action
           d. It is much more convenient for the nurse            will be to:
                                                                       a. Set and turn on the alarm of the
SITUATION: A Client was rushed to the emergency room                       oximeter
and you are his attending nurse. You are performing a                  b. Do nothing since there is no identified
vital sign assessment.                                                     problem
                                                                       c. Cover the fingertip sensor with a towel
11.    All of the following are correct methods in                         or bedsheet
       assessment of the blood pressure EXCEPT:                        d. Change the location of the sensor every
            a. Take the blood pressure reading on both                     four hours
                arms for comparison
            b. Listen to and identify the phases of         16.   The nurse finds it necessary to recheck the blood
                Korotkoff’s sound                                 pressure reading. In case of such re assessment,
            c. Pump the cuff to around 50 mmHg                    the nurse should wait for a period of:
                above the point where the pulse is                    a. 15 seconds
                obliterated                                           b. 1 to 2 minutes
            d. Observe procedures for infection control               c. 30 minutes
                                                                      d. 15 minutes
12.    You attached a pulse oximeter to the client. You
       know that the purpose is to:                         17.   If the arm is said to be elevated when taking the
          a. Determine if the client’s hemoglobin                 blood pressure, it will create a:
               level is low and if he needs blood                      a. False high reading
               transfusion                                             b. False low reading
          b. Check level of client’s tissue perfusion                  c. True false reading
          c. Measure the efficacy of the client’s anti-                d. Indeterminate
               hypertensive medications
          d. Detect oxygen saturation of arterial           18.   You are to assessed the temperature of the
               blood before symptoms of hypoxemia                 client the next morning and found out that he
               develops                                           ate ice cream. How many minutes should you
                                                                  wait before assessing the client’s oral
13.    After a few hours in the Emergency Room, The               temperature?
       client is admitted to the ward with an order of                a. 10 minutes
       hourly monitoring of blood pressure. The nurse                 b. 20 minutes
       finds that the cuff is too narrow and this will                c. 30 minutes
       cause the blood pressure reading to be:                        d. 15 minutes
           a. inconsistent
           b. low systolic and high diastolic               19.   When auscultating the client’s blood pressure
           c. higher than what the reading should be              the nurse hears the following: From 150 mmHg
           d. lower than what the reading should be               to 130 mmHg: Silence, Then: a thumping sound
                                                                  continuing down to 100 mmHg; muffled sound
14.    Through the client’s health history, you gather            continuing down to 80 mmHg and then silence.
What is the client’s blood pressure?                          to lungs. This can be avoided by:
          a. 130/80                                                        a. Cleaning teeth and mouth with cotton
          b. 150/100                                                          swabs soaked with mouthwash to avoid
          c. 100/80                                                           rinsing the buccal cavity
          d. 150/100                                                       b. swabbing the inside of the cheeks and
                                                                              lips, tongue and gums with dry cotton
20.     In a client with a previous blood pressure of                         swabs
        130/80 4 hours ago, how long will it take to                       c. use fingers wrapped with wet cotton
        release the blood pressure cuff to obtain an                          washcloth to rub inside the cheeks,
        accurate reading?                                                     tongue, lips and ums
             a. 10-20 seconds                                              d. suctioning as needed while cleaning the
             b. 30-45 seconds                                                 buccal cavity
             c. 1-1.5 minutes
             d. 3-3.5 minutes                                 25.     Your client has difficulty of breathing and is
                                                                      mouth breathing most of the time. This causes
Situation: Oral care is an important part of hygienic                 dryness of the mouth with unpleasant odor. Oral
practices and promoting client comfort.                               hygiene is recommended for the client and in
                                                                      addition, you will keep the mouth moistened by
21.     An elderly client, 84 years old, is unconscious.              using:
        Assessment of the mouth reveals excessive                         a. salt solution
        dryness and presence of sores. Which of the                       b. petroleum jelly
        following is BEST to use for oral care?                           c. water
             a. lemon glycerine                                           d. mentholated ointment
             b. Mineral oil
             c. hydrogen peroxide                             Situation – Ensuring safety before, during and after a
             d. Normal saline solution                        diagnostic procedure is an important responsibility of
                                                              the nurse.
22.     When performing oral care to an unconscious
        client, which of the following is a special           26.     To help Fernan better tolerate the
        consideration to prevent aspiration of fluids into            bronchoscopy, you should instruct him to
        the lungs?                                                    practice which of the following prior to the
            a. Put the client on a sidelying position                 procedure?
                 with head of bed lowered                                 a. Clenching his fist every 2 minutes
            b. Keep the client dry by placing towel                       b. Breathing in and out through the nose
                 under the chin                                               with his mouth open
            c. Wash hands and observes appropriate                        c. Tensing the shoulder muscles while lying
                 infection control                                            on his back
            d. Clean mouth with oral swabs in a careful                   d. Holding his breath periodically for 30
                 and an orderly progression                                   seconds

23.     The advantages of oral care for a client include      27.     Following a bronchoscopy, which of the
        all of the following, EXCEPT:                                 following complains to Fernan should be noted
             a. decreases bacteria in the mouth and                   as a possible complication:
                 teeth                                                     a. Nausea and vomiting
             b. reduces need to use commercial                             b. Shortness of breath and laryngeal
                 mouthwash which irritate the buccal                          stridor
                 mucosa                                                    c. Blood tinged sputum and coughing
             c. improves client’s appearance and self-                     d. Sore throat and hoarseness
                 confidence
             d. improves appetite and taste of food           28.     Immediately after bronchoscopy, you instructed
                                                                      Fernan to:
24.     A possible problem while providing oral care to                   a. Exercise the neck muscles
        unconscious clients is the risk of fluid aspiration               b. Refrain from coughing and talking

6
7

           c. Breathe deeply                                              d. Weber’s test
           d. Clear his throat
                                                               34.    A nurse is reviewing the arterial blood gas values
29.    Thoracentesis may be performed for cytologic                   of a client and notes that the ph is 7.31, Pco2 is
       study of pleural fluid. As a nurse your most                   50 mmHg, and the bicarbonate is 27 mEq/L. The
       important function during the procedure is to:                 nurse concludes that which acid base
           a. Keep the sterile equipment from                         disturbance is present in this client?
               contamination                                              a. Respiratory acidosis
           b. Assist the physician                                        b. Metabolic acidosis
           c. Open and close the three-way stopcock                       c. Respiratory alkalosis
           d. Observe the patient’s vital signs                           d. Metabolic alkalosis

30.    Right after thoracentesis, which of the following       35.    Allen’s test checks the patency of the:
       is most appropriate intervention?                                  a. Ulnar artery
           a. Instruct the patient not to cough or deep                   b. Carotid artery
               breathe for two hours                                      c. Radial artery
           b. Observe for symptoms of tightness of                        d. Brachial artery
               chest or bleeding
           c. Place an ice pack to the puncture site           Situation 6: Eileen, 45 years old is admitted to the
           d. Remove the dressing to check for                 hospital with a diagnosis of renal calculi. She is
               bleeding                                        experiencing severe flank pain, nauseated and with a
                                                               temperature of 39 0C.
Situation: Knowledge of the acid-base disturbance and
the functions of the electrolytes is necessary to              36.    Given the above assessment data, the most
determine appropriate intervention and nursing actions.               immediate goal of the nurse would be which of
                                                                      the following?
31.    A client with diabetes milletus has a blood                        a. Prevent urinary complication
       glucose level of 644 mg/dL. The nurse interprets                   b. maintains fluid and electrolytes
       that this client is at most risk for the                           c. Alleviate pain
       development of which type of acid-base                             d. Alleviating nausea
       imbalance?
            a. Respiratory acidosis                            37.    After IVP a renal stone was confirmed, a left
            b. Respiratory alkalosis                                  nephrectomy was done. Her post-operative
            c. Metabolic acidosis                                     order includes “daily urine specimen to be sent
            d. Metabolic alkalosis                                    to the laboratory”. Eileen has a foley catheter
                                                                      attached to a urinary drainage system. How will
32.    In a client in the health care clinic, arterial blood          you collect the urine specimen?
       gas analysis gives the following results: pH 7.48,                 a. remove urine from drainage tube with
       PCO2 32 mmHg, PO2 94 mmHg, HCO3 24 mEq/L.                               sterile needle and syringe and empty
       The nurse interprets that the client has which                          urine from the syringe into the
       acid base disturbance?                                                  specimen container
            a. Respiratory acidosis                                       b. empty a sample urine from the
            b. Metabolic acidosis                                              collecting bag into the specimen
            c. Respiratory alkalosis                                           container
            d. Metabolic alkalosis                                        c. Disconnect the drainage tube from the
                                                                               indwelling catheter and allow urine to
33.    A client has an order for ABG analysis on radial                        flow from catheter into the specimen
       artery specimens. The nurse ensures that which                          container.
       of the following has been performed or tested                      d. Disconnect the drainage from the
       before the ABG specimens are drawn?                                     collecting bag and allow the urine to
            a. Guthrie test                                                    flow from the catheter into the
            b. Romberg’s test                                                  specimen container.
            c. Allen’s test
38.    Where would the nurse tape Eileen’s indwelling              regulation is secreted in the:
       catheter in order to reduce urethral irritation?                a. Thyroid gland
           a. to the patient’s inner thigh                             b. Parathyroid gland
           b. to the patient’ buttocks                                 c. Hypothalamus
           c. to the patient’s lower thigh                             d. Anterior pituitary gland
           d. to the patient lower abdomen
                                                           45.     While Parathormone, a hormone that negates
39.    Which of the following menu is appropriate for              the effect of calcitonin is secreted by the:
       one with low sodium diet?                                       a. Thyroid gland
          a. instant noodles, fresh fruits and ice tea                 b. Parathyroid gland
          b. ham and cheese sandwich, fresh fruits                     c. Hypothalamus
              and vegetables                                           d. Anterior pituitary gland
          c. white chicken sandwich, vegetable
              salad and tea                                Situation: The staff nurse supervisor requests all the staff
          d. canned soup, potato salad, and diet soda      nurses to “brainstorm” and learn ways to instruct
                                                           diabetic clients on self-administration of insulin. She
40.    How will you prevent ascending infection to         wants to ensure that there are nurses available daily to
       Eileen who has an indwelling catheter?              do health education classes.
           a. see to it that the drainage tubing
               touches the level of the urine              46.     The plan of the nurse supervisor is an example of
           b. change he catheter every eight hours                     a. in service education process
           c. see to it that the drainage tubing does                  b. efficient management of human
               not touch the level of the urine                            resources
           d. clean catheter may be used since                         c. increasing human resources
               urethral meatus is not a sterile area                   d. primary prevention

Situation: Hormones are secreted by the various glands     47.     When Mrs. Guevarra, a nurse, delegates aspects
in the body. Basic knowledge of the endocrine system is            of the clients care to the nurse-aide who is an
necessary.                                                         unlicensed staff, Mrs. Guevarra
                                                                       a. makes the assignment to teach the staff
41.    Somatocrinin or the Growth hormone releasing                         member
       hormone is secreted by the:                                     b. is assigning the responsibility to the
          a. Hypothalamus                                                   aide but not the accountability for
          b. Posterior pituitary gland                                      those tasks
          c. Anterior pituitary gland                                  c. does not have to supervise or evaluate
          d. Thyroid gland                                                  the aide
                                                                       d. most know how to perform task
42.    All of the following are secreted by the anterior                    delegated
       pituitary gland except:
            a. Somatotropin/Growth hormone                 48.     Connie, the new nurse, appears tired and
            b. Thyroid stimulating hormone                         sluggish and lacks the enthusiasm she had six
            c. Follicle stimulating hormone                        weeks ago when she started the job. The nurse
            d. Gonadotropin hormone releasing                      supervisor should
                hormone                                                a. empathize with the nurse and listen to
                                                                           her
43.    All of the following hormones are hormones                      b. tell her to take the day off
       secreted by the Posterior pituitary gland except:               c. discuss how she is adjusting to her new
            a. Vasopressin                                                 job
            b. Anti-diuretic hormone                                   d. ask about her family life
            c. Oxytocin
            d. Growth hormone                              49.     Process of formal negotiations of working
                                                                   conditions between a group of registered nurses
44.    Calcitonin, a hormone necessary for calcium                 and employer is

8
9

            a.   grievance                                                d. It should disclose previous diagnosis,
            b.   arbitration                                                 prognosis and alternative treatments
            c.   collective bargaining                                       available for the client
            d.   strike
                                                              55.     Delegation is the process of assigning tasks that
50.     You are attending a certification on                          can be performed by a subordinate. The RN
        cardiopulmonary resuscitation (CPR) offered and               should always be accountable and should not
        required by the hospital employing you. This is               lose his accountability. Which of the following is
            a. professional course towards credits                    a role included in delegation?
            b. in-service education                                       a. The RN must supervise all delegated
            c. advance training                                                tasks
            d. continuing education                                       b. After a task has been delegated, it is no
                                                                               longer a responsibility of the RN
Situation: As a nurse, you are aware that proper                          c. The RN is responsible and accountable
documentation in the patient chart is your responsibility.                     for the delegated task in adjunct with
                                                                               the delegate
51.     Which of the following is not a legally binding                   d. Follow up with a delegated task is
        document but nevertheless very important in                            necessary only if the assistive personnel
        the care of all patients in any health care                            is not trustworthy
        setting?
            a. Bill of rights as provided in the Philippine   Situation: When creating your lesson plan for
                 constitution                                 cerebrovascular disease or STROKE. It is important to
            b. Scope of nursing practice as defined by        include the risk factors of stroke.
                 RA 9173
            c. Board of nursing resolution adopting the       56.     The most important risk factor is:
                 code of ethics                                           a. Cigarette smoking
            d. Patient’s bill of rights                                   b. binge drinking
                                                                          c. Hypertension
52.     A nurse gives a wrong medication to the client.                   d. heredity
        Another nurse employed by the same hospital as
        a risk manager will expect to receive which of        57.     Part of your lesson plan is to talk about etiology
        the following communication?                                  or cause of stroke. The types of stroke based on
             a. Incident report                                       cause are the following EXCEPT:
             b. Nursing kardex                                            a. Embolic stroke
             c. Oral report                                               b. diabetic stroke
             d. Complain report                                           c. Hemorrhagic stroke
                                                                          d. thrombotic stroke
53.     Performing a procedure on a client in the
        absence of an informed consent can lead to            58.     Hemmorhagic stroke occurs suddenly usually
        which of the following charges?                               when the person is active. All are causes of
            a. Fraud                                                  hemorrhage, EXCEPT:
            b. Harassment                                                a. phlebitis
            c. Assault and battery                                       b. damage to blood vessel
            d. Breach of confidentiality                                 c. trauma
                                                                         d. aneurysm
54.     Which of the following is the essence of
        informed consent?                                     59.     The nurse emphasizes that intravenous drug
            a. It should have a durable power of                      abuse carries a high risk of stroke. Which drug is
               attorney                                               closely linked to this?
            b. It should have coverage from an                            a. Amphetamines
               insurance company                                          b. shabu
            c. It should respect the client’s freedom                     c. Cocaine
               from coercion                                              d. Demerol
d. Iron 75 mg/100 ml
60.     A participant in the STROKE class asks what is a
        risk factor of stroke. Your best response is:         65.     Which of the following laboratory test result
             a. “More red blood cells thicken blood                   indicate presence of an infectious process?
                 and make clots more possible.”                           a. Erythrocyte sedimentation rate (ESR) 12
             b. “Increased RBC count is linked to high                        mm/hr
                 cholesterol.”                                            b. White blood cells (WBC) 18,000/mm3
             c. “More red blood cell increases                            c. Iron 90 g/100ml
                 hemoglobin content.”                                     d. Neutrophils 67%
             d. “High RBC count increases blood
                 pressure.”                                   Situation: Pleural effusion is the accumulation of fluid in
                                                              the pleural space. Questions 66 to 70 refer to this.
Situation: Recognition of normal values is vital in
assessment of clients with various disorders.                 66.     Which of the following is a finding that the nurse
                                                                      will be able to assess in a client with Pleural
61.     A nurse is reviewing the laboratory test results              effusion?
        for a client with a diagnosis of severe                            a. Reduced or absent breath sound at the
        dehydration. The nurse would expect the                                base of the lungs, dyspnea, tachpynea
        hematocrit level for this client to be which of the                    and shortness of breath
        following?                                                         b. Hypoxemia, hypercapnea and
             a. 60%                                                            respiratory acidosis
             b. 47%                                                        c. Noisy respiration, crackles, stridor and
             c. 45%                                                            wheezing
             d. 32%                                                        d. Tracheal deviation towards the affected
                                                                               side, increased fremitus and loud breath
62.     A nurse is reviewing the electrolyte results of an                     sounds
        assigned client and notes that the potassium
        level is 5.6 mEq/L. Which of the following would      67.     Thoracentesis is performed to the client with
        the nurse expect to note on the ECG as a result               effusion. The nurse knows that the removal of
        of this laboratory value?                                     fluid should be slow. Rapid removal of fluid in
            a. ST depression                                          thoracentesis might cause:
            b. Prominent U wave                                            a. Pneumothorax
            c. Inverted T wave                                             b. Cardiovascular collapse
            d. Tall peaked T waves                                         c. Pleurisy or Pleuritis
                                                                           d. Hypertension
63.     A nurse is reviewing the electrolyte results of an
        assigned client and notes that the potassium          68.     3 Days after thoracentesis, the client again
        level is 3.2 mEq/L. Which of the following would              exhibited respiratory distress. The nurse will
        the nurse expect to note on the ECG as a result               know that pleural effusion has reoccurred when
        of this laboratory value?                                     she noticed a sharp stabbing pain during
            a. U waves                                                inspiration. The physician ordered a closed tube
            b. Elevated T waves                                       thoracotomy for the client. The nurse knows
            c. Absent P waves                                         that the primary function of the chest tube is to:
            d. Elevated ST Segment                                        a. Restore positive intrathoracic pressure
                                                                          b. Restore negative intrathoracic pressure
64.     Dorothy underwent diagnostic test and the                         c. To visualize the intrathoracic content
        result of the blood examination are back. On                      d. As a method of air administration via
        reviewing the result the nurse notices which of                        ventilator
        the following as abnormal finding?
            a. Neutrophils 60%                                69.     The chest tube is functioning properly if:
            b. White blood cells (WBC) 9000/mm                            a. There is an oscillation
            c. Erythrocyte sedimentation rate (ESR) is                    b. There is no bubbling in the drainage
                 39 mm/hr                                                    bottle

10
11

            c. There is a continuous bubbling in the
               waterseal                                    75.     This form of Health Insurance provides
            d. The suction control bottle has a                     comprehensive prepaid health services to
               continuous bubbling                                  enrollees for a fixed periodic payment.
                                                                        a. Health Maintenance Organization
70.     In a client with pleural effusion, the nurse is                 b. Medicare
        instructing appropriate breathing technique.                    c. Philippine Health Insurance Act
        Which of the following is included in the                       d. Hospital Maintenance Organization
        teaching?
             a. Breath normally                             Situation: Nursing ethics is an important part of the
             b. Hold the breath after each inspiration      nursing profession. As the ethical situation arises, so is
                 for 1 full minute                          the need to have an accurate and ethical decision
             c. Practice abdominal breathing                making.
             d. Inhale slowly and hold the breath for 3
                 to 5 seconds after each inhalation         76.     The purpose of having a nurses’ code of ethics is:
                                                                        a. Delineate the scope and areas of nursing
SITUATION: Health care delivery system affects the                         practice
health status of every filipino. As a Nurse, Knowledge of               b. identify nursing action recommended for
this system is expected to ensure quality of life.                         specific health care situations
                                                                        c. To help the public understand
71.     When should rehabilitation commence?                               professional conduct expected of
          a. The day before discharge                                      nurses
          b. When the patient desires                                   d. To define the roles and functions of the
          c. Upon admission                                                health care givers, nurses, clients
          d. 24 hours after discharge
                                                            77.     The principles that govern right and proper
72.     What exemplified the preventive and promotive               conduct of a person regarding life, biology and
        programs in the hospital?                                   the health professionals is referred to as:
           a. Hospital as a center to prevent and                       a. Morality
               control infection                                        b. Religion
           b. Program for smokers                                       c. Values
           c. Program for alcoholics and drug addicts                   d. Bioethics
           d. Hospital Wellness Center
                                                            78.     A subjective feeling about what is right or wrong
73.     Which makes nursing dynamic?                                is said to be:
           a. Every patient is a unique physical,                        a. Morality
               emotional, social and spiritual being                     b. Religion
           b. The patient participate in the overall                     c. Values
               nursing care plan                                         d. Bioethics
           c. Nursing practice is expanding in the light
               of modern developments that takes            79.     Values are said to be the enduring believe about
               place                                                a worth of a person, ideas and belief. If Values
           d. The health status of the patient is                   are going to be a part of a research, this is
               constantly changing and the nurse must               categorized under:
               be cognizant and responsive to these                     a. Qualitative
               changes                                                  b. Experimental
                                                                        c. Quantitative
74.     Prevention is an important responsibility of the                d. Non Experimental
        nurse in:
            a. Hospitals                                    80.     The most important nursing responsibility where
            b. Community                                            ethical situations emerge in patient care is to:
            c. Workplace                                                a. Act only when advised that the action is
            d. All of the above                                              ethically sound
b. Not takes sides, remain neutral and fair
          c. Assume that ethical questions are the        85.    Based on the Code of Ethics for Filipino Nurses,
             responsibility of the health team                   what is regarded as the hallmark of nursing
          d. Be accountable for his or her own                   responsibility and accountability?
             actions                                                 a. Human rights of clients, regardless of
                                                                         creed and gender
81.   Why is there an ethical dilemma?                               b. The privilege of being a registered
        a. the choices involved do not appear to be                      professional nurse
              clearly right or wrong                                 c. Health, being a fundamental right of
        b. a client’s legal right co-exist with the                      every individual
              nurse’s professional obligation                        d. Accurate documentation of actions and
        c. decisions has to be made based on                             outcomes
              societal norms.
        d. decisions has to be mad quickly, often         Situation: As a profession, nursing is dynamic and its
              under stressful conditions                  practice is directed by various theoretical models. To
                                                          demonstrate caring behaviour, the nurse applies various
82.   According to the code of ethics, which of the       nursing models in providing quality nursing care.
      following is the primary responsibility of the
      nurse?                                              86.    When you clean the bedside unit and regularly
           a. Assist towards peaceful death                      attend to the personal hygiene of the patient as
           b. Health is a fundamental right                      well as in washing your hands before and after a
           c. Promotion of health, prevention of                 procedure and in between patients, you indent
              illness, alleviation of suffering and              to facilitate the body’s reparative processes.
              restoration of health                              Which of the following nursing theory are you
           d. Preservation of health at all cost                 applying in the above nursing action?
                                                                     a. Hildegard Peplau
83.   Which of the following is TRUE about the Code                  b. Dorothea Orem
      of Ethics of Filipino Nurses, except:                          c. Virginia Henderson
          a. The Philippine Nurses Association for                   d. Florence Nightingale
               being the accredited professional
               organization was given the privilege to    87.    A communication skill is one of the important
               formulate a Code of Ethics for Nurses             competencies expected of a nurse. Interpersonal
               which the Board of Nursing                        process is viewed as human to human
               promulgated                                       relationship. This statement is an application of
          b. Code for Nurses was first formulated in             whose nursing model?
               1982 published in the Proceedings of the              a. Joyce Travelbee
               Third Annual Convention of the PNA                    b. Martha Rogers
               House of Delegates                                    c. Callista Roy
          c. The present code utilized the Code of                   d. Imogene King
               Good Governance for the Professions in
               the Philippines                            88.    The statement “the health status of an individual
          d. Certificates of Registration of registered          is constantly changing and the nurse must be
               nurses may be revoked or suspended for            cognizant and responsive to these changes” best
               violations of any provisions of the Code          explains which of the following facts about
               of Ethics.                                        nursing?
                                                                      a. Dynamic
84.   Violation of the code of ethics might equate to                 b. Client centred
      the revocation of the nursing license. Who                      c. Holistic
      revokes the license?                                            d. Art
          a. PRC
          b. PNA                                          89.    Virginia Henderson professes that the goal of
          c. DOH                                                 nursing is to work interdependently with other
          d. BON                                                 health care working in assisting the patient to

12
13

        gain independence as quickly as possible. Which                include:
        of the following nursing actions best                              a. Prescription of the doctor to the
        demonstrates this theory in taking care of a 94                         patient’s illness
        year old client with dementia who is totally                       b. Plan of care for patient
        immobile?                                                          c. Patient’s perception of one’s illness
            a. Feeds the patient, brushes his teeth,                       d. Nursing problem and Nursing diagnosis
                 gives the sponge bath
            b. Supervise the watcher in rendering              94.     The medical records that are organized into
                 patient his morning care                              separate section from doctors or nurses has
            c. Put the patient in semi fowler’s position,              more disadvantages than advantages. This is
                 set the over bed table so the patient can             classified as what type of recording?
                 eat by himself, brush his teeth and                       a. POMR
                 sponge himself                                            b. Modified POMR
            d. Assist the patient to turn to his sides and                 c. SOAPIE
                 allow him to brush and feed himself only                  d. SOMR
                 when he feels ready
                                                               95.     Which of the following is the advantage of SOMR
90.     In the self-care deficit theory by Dorothea Orem,              or Traditional recording?
        nursing care becomes necessary when a patient                      a. Increases efficiency in data gathering
        is unable to fulfil his physiological, psychological               b. Reinforces the use of the nursing
        and social needs. A pregnant client needing                            process
        prenatal check-up is classified as:                                c. The caregiver can easily locate proper
             a. Wholly compensatory                                            section for making charting entries
             b. Supportive Educative                                       d. Enhances effective communication
             c. Partially compensatory                                         among health care team members
             d. Non compensatory
                                                               Situation: June is a 24 year old client with symptoms of
Situation: Documentation and reporting are just as             dyspnea, absent breath sounds on the right lung and
important as providing patient care, As such, the nurse        chest x ray revealed pleural effusion. The physician will
must be factual and accurate to ensure quality                 perform thoracentesis.
documentation and reporting.
                                                               96.     Thoracentesis is useful in treating all of the
91.     Health care reports have different purposes. The               following pulmonary disorders except:
        availability of patients’ record to all health team                 a. Hemothorax
        members demonstrates which of the following                         b. Hydrothorax
        purposes:                                                           c. Tuberculosis
            a. Legal documentation                                          d. Empyema
            b. Research
            c. Education                                       97.     Which of the following psychological preparation
            d. Vehicle for communication                               is not relevant for him?
                                                                           a. Telling him that the gauge of the needle
92.     When a nurse commits medication error, she                              and anesthesia to be used
        should accurately document client’s response                       b. Telling him to keep still during the
        and her corresponding action. This is very                              procedure to facilitate the insertion of
        important for which of the following purposes:                          the needle in the correct place
           a. Research                                                     c. Allow June to express his feelings and
           b. Legal documentation                                               concerns
           c. Nursing Audit                                                d. Physician’s explanation on the purpose
           d. Vehicle for communication                                         of the procedure and how it will be done

93.     POMR has been widely used in many teaching             98.     Before thoracentesis, the legal consideration you
        hospitals. One of its unique features is SOAPIE                must check is:
        charting. The P in SOAPIE charting should                          a. Consent is signed by the client
b. Medicine preparation is correct
           c. Position of the client is correct
           d. Consent is signed by relative and
              physician

 99.   As a nurse, you know that the position for June
       before thoracentesis is:
           a. Orthopneic
           b. Low fowlers
           c. Knee-chest
           d. Sidelying position on the affected side

100.   Which of the following anaesthetics drug is used
       for thoracentesis?
            a. Procaine 2%
            b. Demerol 75 mg
            c. Valium 250 mg
            d. Phenobartbital 50 mg




14
15

                                                                      D. Follicle stimulating hormone

NURSING PRACTICE II                                           5. The following month, Mariah suspects she is
                                                                 pregnant. Her urine is positive for Human
Situation: Mariah is a 31 year old lawyer who has been           chorionic gonadotrophin. Which structure
married for 6 months. She consults you for guidance in           produces Hcg?
relation with her menstrual cycle and her desire to get              A. Pituitary gland
pregnant.                                                            B. Trophoblastic cells of the embryo
                                                                     C. Uterine deciduas
    1. She wants to know the length of her menstrual                 D. Ovarian follicles
       cycle. Her previous menstrual period is October
       22 to 26. Her LMB is November 21. Which of the      Situation: Mariah came back and she is now pregnant.
       following number of days will be your correct
       response?                                              6. At 5 month gestation, which of the following
            A. 29                                                fetal development would probably be achieve?
            B. 28                                                    A. Fetal movement are felt by Mariah
            C. 30                                                    B. Vernix caseosa covers the entire body
            D. 31                                                    C. Viable if delivered within this period
                                                                     D. Braxton hicks contractions are observed
    2. You advised her to observe and record the signs
       of Ovulation. Which of the following signs will        7. The nurse palpates the abdomen of Mariah.
       she likely note down?                                     Now At 5 month gestation, What level of the
         1.     A 1 degree Fahrenheit rise in basal body         abdomen can the fundic height be palpated?
                temperature                                          A. Symphysis pubis
         2.     Cervical mucus becomes copious and                   B. Midpoint between the umbilicus and the
                clear                                                   xiphoid process
         3.     One pound increase in weight                         C. Midpoint between the symphysis pubis
         4.     Mittelschmerz                                           and the umbilicus
           A. 1, 2, 4                                                D. Umbilicus
           B. 1, 2, 3
           C. 2, 3, 4                                         8. She worries about her small breasts, thinking
           D. 1, 3, 4                                            that she probably will not be able to breastfeed
                                                                 her baby. Which of the following responses of
    3. You instruct Mariah to keep record of her basal           the nurse is correct?
       temperature every day, which of the following                 A. “The size of your breast will not affect
       instructions is incorrect?                                        your lactation”
           A. If coitus has occurred; this should be                 B. “You can switch to bottle feeding”
                reflected in the chart                               C. “You can try to have exercise to increase
           B. It is best to have coitus on the evening                   the size of your breast”
                following a drop in BBT to become                    D. “Manual expression of milk is possible”
                pregnant
           C. Temperature should be taken                     9. She tells the nurse that she does not take milk
                immediately after waking and before              regularly. She claims that she does not want to
                getting out of bed                               gain too much weight during her pregnancy.
           D. BBT is lowest during the secretory                 Which of the following nursing diagnosis is a
                phase                                            priority?
                                                                     A. Potential self-esteem disturbance
    4. She reports an increase in BBT on December 16.                     related to physiologic changes in
       Which hormone brings about this change in her                      pregnancy
       BBT?                                                          B. Ineffective individual coping related to
           A. Estrogen                                                    physiologic changes in pregnancy
           B. Gonadotropine                                          C. Fear related to the effects of pregnancy
           C. Progesterone                                           D. Knowledge deficit regarding nutritional
requirements of pregnancies related to           15. While talking with Susan, 2 new patients arrived
                 lack of information sources                          and they are covered with large towels and the
                                                                      nurse noticed that there are many cameraman
     10. Which of the following interventions will likely             and news people outside of the OPD. Upon
         ensure compliance of Mariah?                                 assessment the nurse noticed that both of them
            A. Incorporate her food preferences that                  are still nude and the male client’s penis is still
                 are adequately nutritious in her meal                inside the female client’s vagina and the male
                 plan                                                 client said that “I can’t pull it”. Vaginismus was
            B. Consistently counsel toward optimum                    your first impression. You know that The
                 nutritional intake                                   psychological cause of Vaginismus is related to:
            C. Respect her right to reject dietary                        A. The male client inserted the penis too
                 information if she chooses                                     deeply that it stimulates vaginal closure
            D. Inform her of the adverse effects of                       B. The penis was too large that is why the
                 inadequate nutrition to her fetus                              vagina triggered its defense to attempt
                                                                                to close it
Situation: Susan is a patient in the clinic where you work.               C. The vagina does not want to be
She is inquiring about pregnancy.                                               penetrated
                                                                          D. It is due to learning patterns of the
     11. Susan tells you she is worried because she                             female client where she views sex as
         develops breasts later than most of her friends.                       bad or sinful
         Breast development is termed as:
             A. Adrenarche                                    Situation: Overpopulation is one problem in the
             B. Thelarche                                     Philippines that causes economic drain. Most Filipinos
             C. Mamarche                                      are against in legalizing abortion. As a nurse, Mastery of
             D. Menarche                                      contraception is needed to contribute to the society and
                                                              economic growth.
     12. Kevin, Susan’s husband tells you that he is
         considering vasectomy After the birth of their           16. Supposed that Dana, 17 years old, tells you she
         new child. Vasectomy involves the incision of                wants to use fertility awareness method of
         which organ?                                                 contraception. How will she determine her
             A. The testes                                            fertile days?
             B. The epididymis                                             A. She will notice that she feels hot, as if
             C. The vas deferens                                               she has an elevated temperature.
             D. The scrotum                                                B. She should assess whether her cervical
                                                                               mucus is thin, copious, clear and
     13. On examination, Susan has been found of having                        watery.
         a cystocele. A cystocele is:                                      C. She should monitor her emotions for
             A. A sebaceous cyst arising from the vulvar                       sudden anger or crying
                 fold                                                      D. She should assess whether her breasts
             B. Protrusion of intestines into the vagina                       feel sensitive to cool air
             C. Prolapse of the uterus into the vagina
             D. Herniation of the bladder into the                17. Dana chooses to use COC as her family planning
                 vaginal wall                                         method. What is the danger sign of COC you
                                                                      would ask her to report?
     14. Susan typically has menstrual cycle of 34 days.                 A. A stuffy or runny nose
         She told you she had coitus on days 8, 10, 15 and               B. Slight weight gain
         20 of her menstrual cycle. Which is the day on                  C. Arthritis like symptoms
         which she is most likely to conceive?                           D. Migraine headache
             A. 8th day
             B. Day 15                                            18. Dana asks about subcutaneous implants and she
             C. 10th day                                              asks, how long will these implants be effective.
             D. Day 20                                                Your best answer is:
                                                                          A. One month

16
17

           B. Five years
           C. Twelve months                                    23. Another client named Lilia is diagnosed as having
           D. 10 years                                             endometriosis. This condition interferes with
                                                                   fertility because:
   19. Dana asks about female condoms. Which of the                     A. Endometrial implants can block the
       following is true with regards to female                              fallopian tubes
       condoms?                                                         B. The uterine cervix becomes inflamed
            A. The hormone the condom releases                               and swollen
               might cause mild weight gain                             C. The ovaries stop producing adequate
            B. She should insert the condom before                           estrogen
               any penile penetration                                   D. Pressure on the pituitary leads to
            C. She should coat the condom with                               decreased FSH levels
               spermicide before use
            D. Female condoms, unlike male condoms,            24. Lilia is scheduled to have a
               are reusable                                        hysterosalphingogram. Which of the following
                                                                   instructions would you give her regarding this
   20. Dana has asked about GIFT procedure. What                   procedure?
       makes her a good candidate for GIFT?                             A. She will not be able to conceive for 3
          A. She has patent fallopian tubes, so                              months after the procedure
              fertilized ova can be implanted on them                   B. The sonogram of the uterus will reveal
          B. She is RH negative, a necessary                                 any tumors present
              stipulation to rule out RH incompatibility                C. Many women experience mild bleeding
          C. She has normal uterus, so the sperm can                         as an after effect
              be injected through the cervix into it                    D. She may feel some cramping when the
          D. Her husband is taking sildenafil, so all                        dye is inserted
              sperms will be motile
                                                               25. Lilia’s cousin on the other hand, knowing nurse
Situation: Nurse Lorena is a Family Planning and                   Lorena’s specialization asks what artificial
Infertility Nurse Specialist and currently attends to              insemination by donor entails. Which would be
FAMILY PLANNING CLIENTS AND INFERTILE COUPLES.                     your best answer if you were Nurse Lorena?
The following conditions pertain to meeting the nursing                 A. Donor sperm are introduced vaginally
needs of this particular population group.                                   into the uterus or cervix
                                                                        B. Donor sperm are injected intra-
   21. Dina, 17 years old, asks you how a tubal ligation                     abdominally into each ovary
       prevents pregnancy. Which would be the best                      C. Artificial sperm are injected vaginally to
       answer?                                                               test tubal patency
           A. Prostaglandins released from the cut                      D. The husband’s sperm is administered
               fallopian tubes can kill sperm                                intravenously weekly
           B. Sperm cannot enter the uterus because
               the cervical entrance is blocked.           Situation: You are assigned to take care of a group of
           C. Sperm can no longer reach the ova,           patients across the lifespan.
               because the fallopian tubes are blocked
           D. The ovary no longer releases ova as              26. Pain in the elder persons requires careful
               there is nowhere for them to go.                    assessment because they:
                                                                       A. experienced reduce sensory perception
   22. The Dators are a couple undergoing testing for                  B. have increased sensory perception
       infertility. Infertility is said to exist when:                 C. are expected to experience chronic pain
           A. A woman has no uterus                                    D. have a decreased pain threshold
           B. A woman has no children
           C. A couple has been trying to conceive for         27. Administration of analgesics to the older persons
                 1 year                                            requires careful patient assessment because
           D. A couple has wanted a child for 6                    older people:
                 months                                                A. are more sensitive to drugs
B. have increased hepatic, renal and                     D. Chronic poverty
                gastrointestinal function
             C. have increased sensory perception             34. Which of the following signs and symptoms
             D. mobilize drugs more rapidly                       would you most likely find when assessing and
                                                                  infant with Arnold-Chiari malformation?
     28. The elderly patient is at higher risk for urinary            A. Weakness of the leg muscles, loss of
         incontinence because of:                                         sensation in the legs, and restlessness
             A. increased glomerular filtration                       B. Difficulty swallowing, diminished or
             B. decreased bladder capacity                                absent gag reflex, and respiratory
             C. diuretic use                                              distress
             D. dilated urethra                                       C. Difficulty sleeping, hypervigilant, and an
                                                                          arching of the back
     29. Which of the following is the MOST COMMON                    D. Paradoxical irritability, diarrhea, and
         sign of infection among the elderly?                             vomiting.
             A. decreased breath sounds with crackles
             B. pain                                          35. A parent calls you and frantically reports that her
             C. fever                                             child has gotten into her famous ferrous sulfate
             D. change in mental status                           pills and ingested a number of these pills. Her
                                                                  child is now vomiting, has bloody diarrhea, and is
     30. Priorities when caring for the elderly trauma            complaining of abdominal pain. You will tell the
         patient:                                                 mother to:
             A. circulation, airway, breathing                         A. Call emergency medical services (EMS)
             B. airway, breathing, disability (neurologic)                  and get the child to the emergency room
             C. disability (neurologic), airway, breathing             B. Relax because these symptoms will pass
             D. airway, breathing, circulation                              and the child will be fine
                                                                       C. Administer syrup of ipecac
     31. Preschoolers are able to see things from which                D. Call the poison control center
         of the following perspectives?
             A. Their peers                                   36. A client says she heard from a friend that you
             B. Their own and their mother’s                      stop having periods once you are on the “pill”.
             C. Their own and their caregivers’                   The most appropriate response would be:
             D. Only their own                                         A. “The pill prevents the uterus from
                                                                           making such endometrial lining, that is
     32. In conflict management, the win-win approach                      why periods may often be scant or
         occurs when:                                                      skipped occasionally.”
             A. There are two conflicts and the parties                B. “If your friend has missed her period,
                  agree to each one                                        she should stop taking the pills and get a
             B. Each party gives in on 50% of the                          pregnancy test as soon as possible.”
                  disagreements making up the conflict                 C. “The pill should cause a normal
             C. Both parties involved are committed to                     menstrual period every month. It
                  solving the conflict                                     sounds like your friend has not been
             D. The conflict is settled out of court so the                taking the pills properly.”
                  legal system and the parties win                     D. “Missed period can be very dangerous
                                                                           and may lead to the formation of
     33. According to the social-interactional perspective                 precancerous cells.”
         of child abuse and neglect, four factors place the
         family members at risk for abuse. These risk         37. The nurse assessing newborn babies and infants
         factors are the family members at risk for abuse.        during their hospital stay after birth will notice
         These risk factors are the family itself, the            which of the following symptoms as a primary
         caregiver, the child, and                                manifestation of Hirschsprung’s disease?
             A. The presence of a family crisis                       A. A fine rash over the trunk
             B. The national emphasis on sex                          B. Failure to pass meconium during the
             C. Genetics                                                  first 24 to 48 hours after birth

18
19

       C. The skin turns yellow and then brown                    release
          over the first 48 hours of life                      B. a woman is less able to keep the
       D. High-grade fever                                        laceration clean because of her fatigue
                                                               C. healing is limited during pregnancy so
38. A client is 7 months pregnant and has just been               these will not heal until after birth
    diagnosed as having a partial placenta previa.             D. increased bleeding can occur from
    She is stable and has minimal spotting and is                 uterine pressure on leg veins
    being sent home. Which of these instructions to
    the client may indicate a need for further         43. In working with the caregivers of a client with an
    teaching?                                              acute or chronic illness, the nurse would:
         A. Maintain bed rest with bathroom                    A. Teach care daily and let the caregivers
             privileges                                            do a return demonstration just before
         B. Avoid intercourse for three days.                      discharge
         C. Call if contractions occur.                        B. Difficulty swallowing, diminished or
         D. Stay on left side as much as possible                  absent gag reflex, and respiratory
             when lying down.                                      distress.
                                                               C. Difficulty sleeping, hypervigilant, and an
39. A woman has been rushed to the hospital with                   arching of the back
    ruptured membrane. Which of the following                  D. Paradoxical irritability, diarrhea, and
    should the nurse check first?                                  vomiting
        A. Check for the presence of infection
        B. Assess for Prolapse of the umbilical        44. Which of the following roles BEST exemplifies
            cord                                           the expanded role of the nurse?
        C. Check the maternal heart rate                       A. Circulating nurse in surgery
        D. Assess the color of the amniotic fluid              B. Medication nurse
                                                               C. Obstetrical nurse
40. The nurse notes that the infant is wearing a               D. Pediatric nurse practitioner
    plastic-coated diaper. If a topical medication
    were to be prescribed and it were to go on the     45. According to DeRosa and Kochura’s (2006)
    stomachs or buttocks, the nurse would teach the        article entitled “Implement Culturally Competent
    caregivers to:                                         Health Care in your work place,” cultures have
        A. avoid covering the area of the topical          different patterns of verbal and nonverbal
             medication with the diaper                    communication. Which difference does?
        B. avoid the use of clothing on top of the              A. NOT necessarily belong?
             diaper                                             B. Personal behavior
        C. put the diaper on as usual                           C. Subject matter
        D. apply an icepack for 5 minutes to the                D. Eye contact
             outside of the diaper                              E. Conversational style

41. Which of the following factors is most important   46. You are the nurse assigned to work with a child
    in determining the success of relationships used       with acute glomerulonephritis. By following the
    in delivering nursing care?                            prescribed treatment regimen, the child
        A. Type of illness of the client                   experiences a remission. You are now checking
        B. Transference and counter transference           to make sure the child does not have a relapse.
        C. Effective communication                         Which finding would most lead you to the
        D. Personality of the participants                 conclusion that a relapse is happening?
                                                               A. Elevated temperature, cough, sore
42. Grace sustained a laceration on her leg from                   throat, changing complete blood count
    automobile accident. Why are lacerations of                    (CBC) with diiferential
    lower extremities potentially more serious                 B. A urine dipstick measurement of 2+
    among pregnant women than other?                               proteinuria or more for 3 days, or the
        A. lacerations can provoke allergic                        child found to have 3-4+ proteinutria
            responses due to gonadotropic hormone                  plus edema.
C. The urine dipstick showing glucose in the
                urine for 3 days, extreme thirst, increase         51. If a child with diarrhea registers two signs in the
                in urine output, and a moon face.                      yellow row in the IMCI chart, we can classify the
             D. A temperature of 37.8 degrees (100                     patient as:
                degrees F), flank pain, burning                             A. Moderate dehydration
                frequency, urgency on voiding, and                          B. Severe dehydration
                cloudy urine.                                               C. Some dehydration
                                                                            D. No dehydration
     47. The nurse is working with an adolescent who
         complains of being lonely and having a lack of            52. Celeste has had diarrhea for 8 days. There is no
         fulfillment in her life. This adolescent shies away           blood in the stool, he is irritable, his eyes are
         from intimate relationships at times yet at other             sunken, the nurse offers fluid to Celeste and he
         times she appears promiscuous. The nurse will                 drinks eagerly. When the nurse pinched the
         likely work with this adolescent in which of the              abdomen it goes back slowly. How will you
         following areas?                                              classify Celeste’s illness?
              A. Isolation                                                 A. Moderate dehydration
              B. Lack of fulfillment                                       B. Severe dehydration
              C. Loneliness                                                C. Some dehydration
              D. Identity                                                  D. No dehydration

     48. The use of interpersonal decision making,                 53. A child who is 7 weeks has had diarrhea for 14
         psychomotor skills, and application of                        days but has no sign of dehydration is classified
         knowledge expected in the role of a licensed                  as:
         health care professional in the context of public                 A. Persistent diarrhea
         health welfare and safety is an example of:                       B. Dysentery
             A. Delegation                                                 C. Severe dysentery
             B. Responsibility                                             D. Severe persistent diarrhea
             C. Supervision
             D. Competence                                         54. The child with no dehydration needs home
                                                                       treatment. Which of the following is not
     49. The painful phenomenon known as “back labor”                  included in the rules for home treatment in this
         occurs in a client whose fetus in what position?              case?
             A. Brow position                                              A. Forced fluids
             B. Breech position                                            B. When to return
             C. Right Occipito-Anterior Position                           C. Give vitamin A supplement
             D. Left Occipito-Posterior Position                           D. Feeding more

     50. FOCUS methodology stands for:                             55. Fever as used in IMCI includes:
            A. Focus, Organize, Clarify, Understand                       A. Axillary temperature of 37.5 or higher
               and Solution                                               B. Rectal temperature of 38 or higher
            B. Focus, Opportunity, Continuous, Utilize,                   C. Feeling hot to touch
               Substantiate                                               D. All of the above
            C. Focus, Organize, Clarify, Understand,                      E. A and C only
               Substantiate
            D. Focus, Opportunity, Continuous                  Situation: Prevention of Dengue is an important nursing
               (process), Understand, Solution                 responsibility and controlling it’s spread is a priority once
                                                               outbreak has been observed.
SITUATION: The infant and child mortality rate in the low
to middle income countries is ten times higher than                56. An important role of the community health
industrialized countries. In response to this, the WHO                 nurse in the prevention and control of Dengue
and UNICEF launched the protocol Integrated                            H-fever includes:
Management of Childhood Illnesses to reduce the                            A. Advising the elimination of vectors by
morbidity and mortality against childhood illnesses.                           keeping water containers covered

20
21

           B. Conducting strong health education                  health worker should first:
              drives/campaign directed towards                        A. Identify the myths and misconceptions
              proper garbage disposal                                     prevailing in the community
           C. Explaining to the individuals, families,                B. Identify the source of these myths and
              groups and community the nature of                          misconceptions
              the disease and its causation                           C. Explain how and why these myths came
           D. Practicing residual spraying with                           about
              insecticides                                            D. Select the appropriate IEC strategies to
                                                                          correct them
   57. Community health nurses should be alert in
       observing a Dengue suspect. The following is           62. How many percent of measles are prevented by
       NOT an indicator for hospitalization of H-fever            immunization at 9 months of age?
       suspects?                                                     A. 80%
           A. Marked anorexia, abdominal pain and                    B. 99%
               vomiting                                              C. 90%
           B. Increasing hematocrit count                            D. 95%
           C. Cough of 30 days
           D. Persistent headache                             63. After TT3 vaccination a mother is said to be
                                                                  protected to tetanus by around:
   58. The community health nurses’ primary concern                   A. 80%
       in the immediate control of hemorrhage among                   B. 99%
       patients with dengue is:                                       C. 85%
            A. Advising low fiber and non-fat diet                    D. 90%
            B. Providing warmth through light weight
                covers                                        64. If ever convulsions occur after administering
            C. Observing closely the patient for vital            DPT, what should the nurse best suggest to the
                signs leading to shock                            mother?
            D. Keeping the patient at rest                             A. Do not continue DPT vaccination
                                                                           anymore
   59. Which of these signs may NOT be REGARDED as                     B. Advise mother to comeback after 1 week
       a truly positive signs indicative of Dengue H-                  C. Give DT instead of DPT
       fever?                                                          D. Give pertussis of the DPT and remove DT
           A. Prolonged bleeding time
           B. Appearance of at least 20 petechiae             65. These vaccines are given 3 doses at one month
                within 1cm square                                 intervals:
           C. Steadily increasing hematocrit count                    A. DPT, BCG, TT
           D. Fall in the platelet count                              B. OPV, HEP. B, DPT
                                                                      C. DPT, TT, OPV
   60. Which of the following is the most important                   D. Measles, OPV, DPT
       treatment of patients with Dengue H-fever?
           A. Give aspirin for fever                      Situation – With the increasing documented cases of
           B. Replacement of body fluids                  CANCER the best alternative to treatment still remains to
           C. Avoid unnecessary movement of patient       be PREVENTION. The following conditions apply.
           D. Ice cap over the abdomen in case of
              melena                                          66. Which among the following is the primary focus
                                                                  of prevention of cancer?
Situation: Health education and Health promotion is an                A. Elimination of conditions causing cancer
important part of nursing responsibility in the                       B. Diagnosis and treatment
community. Immunization is a form of health promotion                 C. Treatment at early stage
that aims at preventing the common childhood illnesses.               D. Early detection

   61. In correcting misconceptions and myths about           67. In the prevention and control of cancer, which of
       certain diseases and their management, the                 the following activities is the most important
comprehensive nursing board exam reviewer
comprehensive nursing board exam reviewer
comprehensive nursing board exam reviewer
comprehensive nursing board exam reviewer
comprehensive nursing board exam reviewer
comprehensive nursing board exam reviewer
comprehensive nursing board exam reviewer
comprehensive nursing board exam reviewer
comprehensive nursing board exam reviewer
comprehensive nursing board exam reviewer
comprehensive nursing board exam reviewer
comprehensive nursing board exam reviewer
comprehensive nursing board exam reviewer
comprehensive nursing board exam reviewer
comprehensive nursing board exam reviewer
comprehensive nursing board exam reviewer
comprehensive nursing board exam reviewer
comprehensive nursing board exam reviewer
comprehensive nursing board exam reviewer
comprehensive nursing board exam reviewer
comprehensive nursing board exam reviewer
comprehensive nursing board exam reviewer
comprehensive nursing board exam reviewer
comprehensive nursing board exam reviewer
comprehensive nursing board exam reviewer
comprehensive nursing board exam reviewer
comprehensive nursing board exam reviewer
comprehensive nursing board exam reviewer
comprehensive nursing board exam reviewer
comprehensive nursing board exam reviewer
comprehensive nursing board exam reviewer
comprehensive nursing board exam reviewer
comprehensive nursing board exam reviewer
comprehensive nursing board exam reviewer
comprehensive nursing board exam reviewer
comprehensive nursing board exam reviewer
comprehensive nursing board exam reviewer
comprehensive nursing board exam reviewer
comprehensive nursing board exam reviewer
comprehensive nursing board exam reviewer
comprehensive nursing board exam reviewer
comprehensive nursing board exam reviewer
comprehensive nursing board exam reviewer
comprehensive nursing board exam reviewer
comprehensive nursing board exam reviewer
comprehensive nursing board exam reviewer
comprehensive nursing board exam reviewer
comprehensive nursing board exam reviewer
comprehensive nursing board exam reviewer
comprehensive nursing board exam reviewer
comprehensive nursing board exam reviewer
comprehensive nursing board exam reviewer
comprehensive nursing board exam reviewer
comprehensive nursing board exam reviewer
comprehensive nursing board exam reviewer
comprehensive nursing board exam reviewer
comprehensive nursing board exam reviewer
comprehensive nursing board exam reviewer
comprehensive nursing board exam reviewer
comprehensive nursing board exam reviewer
comprehensive nursing board exam reviewer
comprehensive nursing board exam reviewer
comprehensive nursing board exam reviewer
comprehensive nursing board exam reviewer
comprehensive nursing board exam reviewer
comprehensive nursing board exam reviewer
comprehensive nursing board exam reviewer
comprehensive nursing board exam reviewer
comprehensive nursing board exam reviewer
comprehensive nursing board exam reviewer
comprehensive nursing board exam reviewer
comprehensive nursing board exam reviewer
comprehensive nursing board exam reviewer
comprehensive nursing board exam reviewer
comprehensive nursing board exam reviewer
comprehensive nursing board exam reviewer
comprehensive nursing board exam reviewer
comprehensive nursing board exam reviewer
comprehensive nursing board exam reviewer
comprehensive nursing board exam reviewer
comprehensive nursing board exam reviewer
comprehensive nursing board exam reviewer
comprehensive nursing board exam reviewer
comprehensive nursing board exam reviewer
comprehensive nursing board exam reviewer
comprehensive nursing board exam reviewer
comprehensive nursing board exam reviewer
comprehensive nursing board exam reviewer
comprehensive nursing board exam reviewer
comprehensive nursing board exam reviewer
comprehensive nursing board exam reviewer
comprehensive nursing board exam reviewer
comprehensive nursing board exam reviewer
comprehensive nursing board exam reviewer
comprehensive nursing board exam reviewer
comprehensive nursing board exam reviewer
comprehensive nursing board exam reviewer
comprehensive nursing board exam reviewer
comprehensive nursing board exam reviewer
comprehensive nursing board exam reviewer
comprehensive nursing board exam reviewer
comprehensive nursing board exam reviewer
comprehensive nursing board exam reviewer
comprehensive nursing board exam reviewer
comprehensive nursing board exam reviewer
comprehensive nursing board exam reviewer
comprehensive nursing board exam reviewer
comprehensive nursing board exam reviewer
comprehensive nursing board exam reviewer
comprehensive nursing board exam reviewer
comprehensive nursing board exam reviewer
comprehensive nursing board exam reviewer
comprehensive nursing board exam reviewer
comprehensive nursing board exam reviewer
comprehensive nursing board exam reviewer
comprehensive nursing board exam reviewer
comprehensive nursing board exam reviewer
comprehensive nursing board exam reviewer
comprehensive nursing board exam reviewer
comprehensive nursing board exam reviewer
comprehensive nursing board exam reviewer
comprehensive nursing board exam reviewer
comprehensive nursing board exam reviewer
comprehensive nursing board exam reviewer
comprehensive nursing board exam reviewer
comprehensive nursing board exam reviewer
comprehensive nursing board exam reviewer
comprehensive nursing board exam reviewer
comprehensive nursing board exam reviewer
comprehensive nursing board exam reviewer
comprehensive nursing board exam reviewer
comprehensive nursing board exam reviewer
comprehensive nursing board exam reviewer
comprehensive nursing board exam reviewer
comprehensive nursing board exam reviewer
comprehensive nursing board exam reviewer
comprehensive nursing board exam reviewer
comprehensive nursing board exam reviewer
comprehensive nursing board exam reviewer
comprehensive nursing board exam reviewer
comprehensive nursing board exam reviewer
comprehensive nursing board exam reviewer
comprehensive nursing board exam reviewer
comprehensive nursing board exam reviewer
comprehensive nursing board exam reviewer
comprehensive nursing board exam reviewer
comprehensive nursing board exam reviewer
comprehensive nursing board exam reviewer
comprehensive nursing board exam reviewer
comprehensive nursing board exam reviewer
comprehensive nursing board exam reviewer
comprehensive nursing board exam reviewer
comprehensive nursing board exam reviewer
comprehensive nursing board exam reviewer
comprehensive nursing board exam reviewer
comprehensive nursing board exam reviewer
comprehensive nursing board exam reviewer
comprehensive nursing board exam reviewer
comprehensive nursing board exam reviewer
comprehensive nursing board exam reviewer
comprehensive nursing board exam reviewer
comprehensive nursing board exam reviewer
comprehensive nursing board exam reviewer
comprehensive nursing board exam reviewer
comprehensive nursing board exam reviewer
comprehensive nursing board exam reviewer
comprehensive nursing board exam reviewer
comprehensive nursing board exam reviewer
comprehensive nursing board exam reviewer
comprehensive nursing board exam reviewer
comprehensive nursing board exam reviewer
comprehensive nursing board exam reviewer
comprehensive nursing board exam reviewer
comprehensive nursing board exam reviewer
comprehensive nursing board exam reviewer
comprehensive nursing board exam reviewer
comprehensive nursing board exam reviewer
comprehensive nursing board exam reviewer
comprehensive nursing board exam reviewer
comprehensive nursing board exam reviewer
comprehensive nursing board exam reviewer
comprehensive nursing board exam reviewer
comprehensive nursing board exam reviewer
comprehensive nursing board exam reviewer
comprehensive nursing board exam reviewer
comprehensive nursing board exam reviewer
comprehensive nursing board exam reviewer
comprehensive nursing board exam reviewer
comprehensive nursing board exam reviewer
comprehensive nursing board exam reviewer
comprehensive nursing board exam reviewer
comprehensive nursing board exam reviewer
comprehensive nursing board exam reviewer
comprehensive nursing board exam reviewer
comprehensive nursing board exam reviewer
comprehensive nursing board exam reviewer
comprehensive nursing board exam reviewer
comprehensive nursing board exam reviewer
comprehensive nursing board exam reviewer
comprehensive nursing board exam reviewer
comprehensive nursing board exam reviewer
comprehensive nursing board exam reviewer
comprehensive nursing board exam reviewer
comprehensive nursing board exam reviewer
comprehensive nursing board exam reviewer
comprehensive nursing board exam reviewer
comprehensive nursing board exam reviewer
comprehensive nursing board exam reviewer
comprehensive nursing board exam reviewer
comprehensive nursing board exam reviewer
comprehensive nursing board exam reviewer
comprehensive nursing board exam reviewer
comprehensive nursing board exam reviewer
comprehensive nursing board exam reviewer
comprehensive nursing board exam reviewer
comprehensive nursing board exam reviewer
comprehensive nursing board exam reviewer
comprehensive nursing board exam reviewer
comprehensive nursing board exam reviewer
comprehensive nursing board exam reviewer
comprehensive nursing board exam reviewer
comprehensive nursing board exam reviewer
comprehensive nursing board exam reviewer
comprehensive nursing board exam reviewer
comprehensive nursing board exam reviewer
comprehensive nursing board exam reviewer
comprehensive nursing board exam reviewer
comprehensive nursing board exam reviewer
comprehensive nursing board exam reviewer
comprehensive nursing board exam reviewer
comprehensive nursing board exam reviewer
comprehensive nursing board exam reviewer
comprehensive nursing board exam reviewer
comprehensive nursing board exam reviewer
comprehensive nursing board exam reviewer
comprehensive nursing board exam reviewer
comprehensive nursing board exam reviewer
comprehensive nursing board exam reviewer
comprehensive nursing board exam reviewer
comprehensive nursing board exam reviewer
comprehensive nursing board exam reviewer
comprehensive nursing board exam reviewer
comprehensive nursing board exam reviewer
comprehensive nursing board exam reviewer
comprehensive nursing board exam reviewer
comprehensive nursing board exam reviewer
comprehensive nursing board exam reviewer
comprehensive nursing board exam reviewer
comprehensive nursing board exam reviewer
comprehensive nursing board exam reviewer
comprehensive nursing board exam reviewer
comprehensive nursing board exam reviewer
comprehensive nursing board exam reviewer
comprehensive nursing board exam reviewer
comprehensive nursing board exam reviewer
comprehensive nursing board exam reviewer
comprehensive nursing board exam reviewer
comprehensive nursing board exam reviewer
comprehensive nursing board exam reviewer
comprehensive nursing board exam reviewer
comprehensive nursing board exam reviewer
comprehensive nursing board exam reviewer
comprehensive nursing board exam reviewer
comprehensive nursing board exam reviewer
comprehensive nursing board exam reviewer
comprehensive nursing board exam reviewer
comprehensive nursing board exam reviewer
comprehensive nursing board exam reviewer
comprehensive nursing board exam reviewer
comprehensive nursing board exam reviewer
comprehensive nursing board exam reviewer
comprehensive nursing board exam reviewer
comprehensive nursing board exam reviewer
comprehensive nursing board exam reviewer
comprehensive nursing board exam reviewer
comprehensive nursing board exam reviewer
comprehensive nursing board exam reviewer
comprehensive nursing board exam reviewer
comprehensive nursing board exam reviewer
comprehensive nursing board exam reviewer
comprehensive nursing board exam reviewer
comprehensive nursing board exam reviewer
comprehensive nursing board exam reviewer
comprehensive nursing board exam reviewer
comprehensive nursing board exam reviewer
comprehensive nursing board exam reviewer
comprehensive nursing board exam reviewer
comprehensive nursing board exam reviewer
comprehensive nursing board exam reviewer
comprehensive nursing board exam reviewer
comprehensive nursing board exam reviewer
comprehensive nursing board exam reviewer
comprehensive nursing board exam reviewer
comprehensive nursing board exam reviewer
comprehensive nursing board exam reviewer
comprehensive nursing board exam reviewer
comprehensive nursing board exam reviewer
comprehensive nursing board exam reviewer
comprehensive nursing board exam reviewer
comprehensive nursing board exam reviewer
comprehensive nursing board exam reviewer
comprehensive nursing board exam reviewer
comprehensive nursing board exam reviewer
comprehensive nursing board exam reviewer
comprehensive nursing board exam reviewer
comprehensive nursing board exam reviewer
comprehensive nursing board exam reviewer
comprehensive nursing board exam reviewer
comprehensive nursing board exam reviewer
comprehensive nursing board exam reviewer
comprehensive nursing board exam reviewer
comprehensive nursing board exam reviewer
comprehensive nursing board exam reviewer
comprehensive nursing board exam reviewer
comprehensive nursing board exam reviewer
comprehensive nursing board exam reviewer
comprehensive nursing board exam reviewer
comprehensive nursing board exam reviewer
comprehensive nursing board exam reviewer
comprehensive nursing board exam reviewer
comprehensive nursing board exam reviewer
comprehensive nursing board exam reviewer
comprehensive nursing board exam reviewer
comprehensive nursing board exam reviewer
comprehensive nursing board exam reviewer
comprehensive nursing board exam reviewer
comprehensive nursing board exam reviewer
comprehensive nursing board exam reviewer
comprehensive nursing board exam reviewer
comprehensive nursing board exam reviewer
comprehensive nursing board exam reviewer
comprehensive nursing board exam reviewer
comprehensive nursing board exam reviewer
comprehensive nursing board exam reviewer
comprehensive nursing board exam reviewer
comprehensive nursing board exam reviewer
comprehensive nursing board exam reviewer
comprehensive nursing board exam reviewer
comprehensive nursing board exam reviewer
comprehensive nursing board exam reviewer
comprehensive nursing board exam reviewer
comprehensive nursing board exam reviewer
comprehensive nursing board exam reviewer
comprehensive nursing board exam reviewer
comprehensive nursing board exam reviewer
comprehensive nursing board exam reviewer
comprehensive nursing board exam reviewer
comprehensive nursing board exam reviewer
comprehensive nursing board exam reviewer
comprehensive nursing board exam reviewer
comprehensive nursing board exam reviewer
comprehensive nursing board exam reviewer
comprehensive nursing board exam reviewer
comprehensive nursing board exam reviewer
comprehensive nursing board exam reviewer
comprehensive nursing board exam reviewer
comprehensive nursing board exam reviewer
comprehensive nursing board exam reviewer
comprehensive nursing board exam reviewer
comprehensive nursing board exam reviewer
comprehensive nursing board exam reviewer
comprehensive nursing board exam reviewer
comprehensive nursing board exam reviewer
comprehensive nursing board exam reviewer
comprehensive nursing board exam reviewer
comprehensive nursing board exam reviewer
comprehensive nursing board exam reviewer
comprehensive nursing board exam reviewer
comprehensive nursing board exam reviewer
comprehensive nursing board exam reviewer
comprehensive nursing board exam reviewer
comprehensive nursing board exam reviewer
comprehensive nursing board exam reviewer
comprehensive nursing board exam reviewer
comprehensive nursing board exam reviewer
comprehensive nursing board exam reviewer
comprehensive nursing board exam reviewer
comprehensive nursing board exam reviewer
comprehensive nursing board exam reviewer
comprehensive nursing board exam reviewer
comprehensive nursing board exam reviewer
comprehensive nursing board exam reviewer
comprehensive nursing board exam reviewer
comprehensive nursing board exam reviewer
comprehensive nursing board exam reviewer
comprehensive nursing board exam reviewer
comprehensive nursing board exam reviewer
comprehensive nursing board exam reviewer
comprehensive nursing board exam reviewer
comprehensive nursing board exam reviewer
comprehensive nursing board exam reviewer
comprehensive nursing board exam reviewer
comprehensive nursing board exam reviewer
comprehensive nursing board exam reviewer
comprehensive nursing board exam reviewer
comprehensive nursing board exam reviewer
comprehensive nursing board exam reviewer
comprehensive nursing board exam reviewer
comprehensive nursing board exam reviewer
comprehensive nursing board exam reviewer
comprehensive nursing board exam reviewer
comprehensive nursing board exam reviewer
comprehensive nursing board exam reviewer
comprehensive nursing board exam reviewer
comprehensive nursing board exam reviewer
comprehensive nursing board exam reviewer
comprehensive nursing board exam reviewer
comprehensive nursing board exam reviewer
comprehensive nursing board exam reviewer
comprehensive nursing board exam reviewer
comprehensive nursing board exam reviewer
comprehensive nursing board exam reviewer
comprehensive nursing board exam reviewer
comprehensive nursing board exam reviewer
comprehensive nursing board exam reviewer
comprehensive nursing board exam reviewer
comprehensive nursing board exam reviewer
comprehensive nursing board exam reviewer
comprehensive nursing board exam reviewer
comprehensive nursing board exam reviewer
comprehensive nursing board exam reviewer
comprehensive nursing board exam reviewer
comprehensive nursing board exam reviewer
comprehensive nursing board exam reviewer
comprehensive nursing board exam reviewer
comprehensive nursing board exam reviewer
comprehensive nursing board exam reviewer
comprehensive nursing board exam reviewer
comprehensive nursing board exam reviewer
comprehensive nursing board exam reviewer
comprehensive nursing board exam reviewer
comprehensive nursing board exam reviewer
comprehensive nursing board exam reviewer
comprehensive nursing board exam reviewer
comprehensive nursing board exam reviewer
comprehensive nursing board exam reviewer
comprehensive nursing board exam reviewer
comprehensive nursing board exam reviewer
comprehensive nursing board exam reviewer
comprehensive nursing board exam reviewer
comprehensive nursing board exam reviewer
comprehensive nursing board exam reviewer
comprehensive nursing board exam reviewer
comprehensive nursing board exam reviewer
comprehensive nursing board exam reviewer
comprehensive nursing board exam reviewer
comprehensive nursing board exam reviewer
comprehensive nursing board exam reviewer
comprehensive nursing board exam reviewer
comprehensive nursing board exam reviewer
comprehensive nursing board exam reviewer
comprehensive nursing board exam reviewer
comprehensive nursing board exam reviewer
comprehensive nursing board exam reviewer
comprehensive nursing board exam reviewer
comprehensive nursing board exam reviewer
comprehensive nursing board exam reviewer
comprehensive nursing board exam reviewer
comprehensive nursing board exam reviewer
comprehensive nursing board exam reviewer
comprehensive nursing board exam reviewer
comprehensive nursing board exam reviewer
comprehensive nursing board exam reviewer
comprehensive nursing board exam reviewer
comprehensive nursing board exam reviewer
comprehensive nursing board exam reviewer
comprehensive nursing board exam reviewer
comprehensive nursing board exam reviewer
comprehensive nursing board exam reviewer
comprehensive nursing board exam reviewer
comprehensive nursing board exam reviewer
comprehensive nursing board exam reviewer
comprehensive nursing board exam reviewer
comprehensive nursing board exam reviewer
comprehensive nursing board exam reviewer
comprehensive nursing board exam reviewer
comprehensive nursing board exam reviewer
comprehensive nursing board exam reviewer
comprehensive nursing board exam reviewer
comprehensive nursing board exam reviewer
comprehensive nursing board exam reviewer
comprehensive nursing board exam reviewer
comprehensive nursing board exam reviewer
comprehensive nursing board exam reviewer
comprehensive nursing board exam reviewer
comprehensive nursing board exam reviewer
comprehensive nursing board exam reviewer
comprehensive nursing board exam reviewer
comprehensive nursing board exam reviewer
comprehensive nursing board exam reviewer
comprehensive nursing board exam reviewer
comprehensive nursing board exam reviewer
comprehensive nursing board exam reviewer
comprehensive nursing board exam reviewer
comprehensive nursing board exam reviewer
comprehensive nursing board exam reviewer
comprehensive nursing board exam reviewer
comprehensive nursing board exam reviewer
comprehensive nursing board exam reviewer
comprehensive nursing board exam reviewer
comprehensive nursing board exam reviewer
comprehensive nursing board exam reviewer
comprehensive nursing board exam reviewer
comprehensive nursing board exam reviewer
comprehensive nursing board exam reviewer
comprehensive nursing board exam reviewer
comprehensive nursing board exam reviewer
comprehensive nursing board exam reviewer
comprehensive nursing board exam reviewer
comprehensive nursing board exam reviewer
comprehensive nursing board exam reviewer
comprehensive nursing board exam reviewer
comprehensive nursing board exam reviewer
comprehensive nursing board exam reviewer
comprehensive nursing board exam reviewer
comprehensive nursing board exam reviewer
comprehensive nursing board exam reviewer
comprehensive nursing board exam reviewer
comprehensive nursing board exam reviewer
comprehensive nursing board exam reviewer
comprehensive nursing board exam reviewer
comprehensive nursing board exam reviewer
comprehensive nursing board exam reviewer
comprehensive nursing board exam reviewer
comprehensive nursing board exam reviewer

Contenu connexe

Tendances

Theories, models, & frameworks
Theories, models, & frameworksTheories, models, & frameworks
Theories, models, & frameworksMinette Din
 
Community health nursing examination part i answer key
Community health nursing examination part i answer keyCommunity health nursing examination part i answer key
Community health nursing examination part i answer keyryanmejia
 
Nursing Informatics historical perspective
Nursing Informatics historical perspectiveNursing Informatics historical perspective
Nursing Informatics historical perspectivejhonee balmeo
 
Emergency nursing questionnaires
Emergency nursing questionnairesEmergency nursing questionnaires
Emergency nursing questionnairesJoan Delgado
 
Culturally competent nursing care - To the Filipino Patient
Culturally competent nursing care - To the Filipino PatientCulturally competent nursing care - To the Filipino Patient
Culturally competent nursing care - To the Filipino Patientmelijaje87
 
Chapter4 ethical issues
Chapter4  ethical issuesChapter4  ethical issues
Chapter4 ethical issuesCath Almonte
 
Nursing informatics: background and application
Nursing informatics: background and applicationNursing informatics: background and application
Nursing informatics: background and applicationjhonee balmeo
 
Imci answer key
Imci answer keyImci answer key
Imci answer keyChisupa
 
IV Fluids Clinical Discussion
IV Fluids Clinical DiscussionIV Fluids Clinical Discussion
IV Fluids Clinical Discussionjhonee balmeo
 

Tendances (20)

Theories, models, & frameworks
Theories, models, & frameworksTheories, models, & frameworks
Theories, models, & frameworks
 
Community health nursing examination part i answer key
Community health nursing examination part i answer keyCommunity health nursing examination part i answer key
Community health nursing examination part i answer key
 
NCM-120-LEC-NOTES.docx
NCM-120-LEC-NOTES.docxNCM-120-LEC-NOTES.docx
NCM-120-LEC-NOTES.docx
 
Copar
CoparCopar
Copar
 
Family Diagnosis *CHN
Family Diagnosis *CHNFamily Diagnosis *CHN
Family Diagnosis *CHN
 
Nursing Informatics historical perspective
Nursing Informatics historical perspectiveNursing Informatics historical perspective
Nursing Informatics historical perspective
 
49821251 ncp
49821251 ncp49821251 ncp
49821251 ncp
 
Nursing code of ethics
Nursing code of ethicsNursing code of ethics
Nursing code of ethics
 
Nursing Care Plan Guide
Nursing Care Plan GuideNursing Care Plan Guide
Nursing Care Plan Guide
 
Emergency nursing questionnaires
Emergency nursing questionnairesEmergency nursing questionnaires
Emergency nursing questionnaires
 
Culturally competent nursing care - To the Filipino Patient
Culturally competent nursing care - To the Filipino PatientCulturally competent nursing care - To the Filipino Patient
Culturally competent nursing care - To the Filipino Patient
 
Chapter4 ethical issues
Chapter4  ethical issuesChapter4  ethical issues
Chapter4 ethical issues
 
Transcribing doctor’s order
Transcribing doctor’s orderTranscribing doctor’s order
Transcribing doctor’s order
 
Diary
DiaryDiary
Diary
 
Copar
CoparCopar
Copar
 
Nursing informatics: background and application
Nursing informatics: background and applicationNursing informatics: background and application
Nursing informatics: background and application
 
Community Health Nursing
Community Health NursingCommunity Health Nursing
Community Health Nursing
 
Family health assessment
Family health assessmentFamily health assessment
Family health assessment
 
Imci answer key
Imci answer keyImci answer key
Imci answer key
 
IV Fluids Clinical Discussion
IV Fluids Clinical DiscussionIV Fluids Clinical Discussion
IV Fluids Clinical Discussion
 

Similaire à comprehensive nursing board exam reviewer

68627164 comprehensive-nursing-board-exam-review-500-page-test-111213225254-p...
68627164 comprehensive-nursing-board-exam-review-500-page-test-111213225254-p...68627164 comprehensive-nursing-board-exam-review-500-page-test-111213225254-p...
68627164 comprehensive-nursing-board-exam-review-500-page-test-111213225254-p...Jhoanna Marie Ong
 
68627164 comprehensive-nursing-board-exam-review-500-page-test-111213225254-p...
68627164 comprehensive-nursing-board-exam-review-500-page-test-111213225254-p...68627164 comprehensive-nursing-board-exam-review-500-page-test-111213225254-p...
68627164 comprehensive-nursing-board-exam-review-500-page-test-111213225254-p...vanblan
 
PreTest Emergency Medicine.pdf
PreTest Emergency Medicine.pdfPreTest Emergency Medicine.pdf
PreTest Emergency Medicine.pdfEidleMohamedsaed
 
Evidence for the ten steps to succesful breastfeeding
Evidence for the ten steps to succesful breastfeedingEvidence for the ten steps to succesful breastfeeding
Evidence for the ten steps to succesful breastfeedingPaul Mark Pilar
 
Evidence for the ten steps to succesful breastfeeding
Evidence for the ten steps to succesful breastfeedingEvidence for the ten steps to succesful breastfeeding
Evidence for the ten steps to succesful breastfeedingPaul Mark Pilar
 
Medicine PreTest Self.pdf
Medicine PreTest Self.pdfMedicine PreTest Self.pdf
Medicine PreTest Self.pdfShriefElghazaly
 
massage-textbook.pdf
massage-textbook.pdfmassage-textbook.pdf
massage-textbook.pdfMehreen601809
 
NCLEX-PN Study Guide
NCLEX-PN Study GuideNCLEX-PN Study Guide
NCLEX-PN Study Guidejconcepcion1
 
Barriers to civility paper
Barriers to civility paperBarriers to civility paper
Barriers to civility paperJenny Erkfitz
 
Manejo de cefaleas jovenes y adultos
Manejo de cefaleas jovenes y adultosManejo de cefaleas jovenes y adultos
Manejo de cefaleas jovenes y adultossilvanaveneros
 
ed_PatientAssessFCS05.pdf
ed_PatientAssessFCS05.pdfed_PatientAssessFCS05.pdf
ed_PatientAssessFCS05.pdfssuserd131ec
 
Final general pharmacology notes by Obin Abraham Olaka, St. Mary's hospital l...
Final general pharmacology notes by Obin Abraham Olaka, St. Mary's hospital l...Final general pharmacology notes by Obin Abraham Olaka, St. Mary's hospital l...
Final general pharmacology notes by Obin Abraham Olaka, St. Mary's hospital l...obin abraham olaka
 

Similaire à comprehensive nursing board exam reviewer (20)

68627164 comprehensive-nursing-board-exam-review-500-page-test-111213225254-p...
68627164 comprehensive-nursing-board-exam-review-500-page-test-111213225254-p...68627164 comprehensive-nursing-board-exam-review-500-page-test-111213225254-p...
68627164 comprehensive-nursing-board-exam-review-500-page-test-111213225254-p...
 
68627164 comprehensive-nursing-board-exam-review-500-page-test-111213225254-p...
68627164 comprehensive-nursing-board-exam-review-500-page-test-111213225254-p...68627164 comprehensive-nursing-board-exam-review-500-page-test-111213225254-p...
68627164 comprehensive-nursing-board-exam-review-500-page-test-111213225254-p...
 
PreTest Emergency Medicine.pdf
PreTest Emergency Medicine.pdfPreTest Emergency Medicine.pdf
PreTest Emergency Medicine.pdf
 
Evidence for the ten steps to succesful breastfeeding
Evidence for the ten steps to succesful breastfeedingEvidence for the ten steps to succesful breastfeeding
Evidence for the ten steps to succesful breastfeeding
 
Evidence for the ten steps to succesful breastfeeding
Evidence for the ten steps to succesful breastfeedingEvidence for the ten steps to succesful breastfeeding
Evidence for the ten steps to succesful breastfeeding
 
WorldWidePE
WorldWidePEWorldWidePE
WorldWidePE
 
Final Thesis Paper
Final Thesis PaperFinal Thesis Paper
Final Thesis Paper
 
Medicine PreTest Self.pdf
Medicine PreTest Self.pdfMedicine PreTest Self.pdf
Medicine PreTest Self.pdf
 
massage-textbook.pdf
massage-textbook.pdfmassage-textbook.pdf
massage-textbook.pdf
 
NCLEX-PN Study Guide
NCLEX-PN Study GuideNCLEX-PN Study Guide
NCLEX-PN Study Guide
 
Health and well being dwp report
Health and well being dwp reportHealth and well being dwp report
Health and well being dwp report
 
CID review
CID reviewCID review
CID review
 
FRONT PAGES
FRONT PAGESFRONT PAGES
FRONT PAGES
 
La84trackfield
La84trackfieldLa84trackfield
La84trackfield
 
IVMS-Basic Medical Terminology-PROGRAMMED INSTRUCTION
IVMS-Basic Medical Terminology-PROGRAMMED INSTRUCTIONIVMS-Basic Medical Terminology-PROGRAMMED INSTRUCTION
IVMS-Basic Medical Terminology-PROGRAMMED INSTRUCTION
 
Barriers to civility paper
Barriers to civility paperBarriers to civility paper
Barriers to civility paper
 
Ebp
EbpEbp
Ebp
 
Manejo de cefaleas jovenes y adultos
Manejo de cefaleas jovenes y adultosManejo de cefaleas jovenes y adultos
Manejo de cefaleas jovenes y adultos
 
ed_PatientAssessFCS05.pdf
ed_PatientAssessFCS05.pdfed_PatientAssessFCS05.pdf
ed_PatientAssessFCS05.pdf
 
Final general pharmacology notes by Obin Abraham Olaka, St. Mary's hospital l...
Final general pharmacology notes by Obin Abraham Olaka, St. Mary's hospital l...Final general pharmacology notes by Obin Abraham Olaka, St. Mary's hospital l...
Final general pharmacology notes by Obin Abraham Olaka, St. Mary's hospital l...
 

Dernier

AORTIC DISSECTION and management of aortic dissection
AORTIC DISSECTION and management of aortic dissectionAORTIC DISSECTION and management of aortic dissection
AORTIC DISSECTION and management of aortic dissectiondrhanifmohdali
 
FDMA FLAP - The first dorsal metacarpal artery (FDMA) flap is used mainly for...
FDMA FLAP - The first dorsal metacarpal artery (FDMA) flap is used mainly for...FDMA FLAP - The first dorsal metacarpal artery (FDMA) flap is used mainly for...
FDMA FLAP - The first dorsal metacarpal artery (FDMA) flap is used mainly for...Shubhanshu Gaurav
 
SGK LEUKEMIA KINH DÒNG BẠCH CÂU HẠT HAY.pdf
SGK LEUKEMIA KINH DÒNG BẠCH CÂU HẠT HAY.pdfSGK LEUKEMIA KINH DÒNG BẠCH CÂU HẠT HAY.pdf
SGK LEUKEMIA KINH DÒNG BẠCH CÂU HẠT HAY.pdfHongBiThi1
 
EXERCISE PERFORMANCE.pptx, Lung function
EXERCISE PERFORMANCE.pptx, Lung functionEXERCISE PERFORMANCE.pptx, Lung function
EXERCISE PERFORMANCE.pptx, Lung functionkrishnareddy157915
 
Physiology of Smooth Muscles -Mechanics of contraction and relaxation
Physiology of Smooth Muscles -Mechanics of contraction and relaxationPhysiology of Smooth Muscles -Mechanics of contraction and relaxation
Physiology of Smooth Muscles -Mechanics of contraction and relaxationMedicoseAcademics
 
PAIN/CLASSIFICATION AND MANAGEMENT OF PAIN.pdf
PAIN/CLASSIFICATION AND MANAGEMENT OF PAIN.pdfPAIN/CLASSIFICATION AND MANAGEMENT OF PAIN.pdf
PAIN/CLASSIFICATION AND MANAGEMENT OF PAIN.pdfDolisha Warbi
 
Trustworthiness of AI based predictions Aachen 2024
Trustworthiness of AI based predictions Aachen 2024Trustworthiness of AI based predictions Aachen 2024
Trustworthiness of AI based predictions Aachen 2024EwoutSteyerberg1
 
CONNECTIVE TISSUE (ANATOMY AND PHYSIOLOGY).pdf
CONNECTIVE TISSUE (ANATOMY AND PHYSIOLOGY).pdfCONNECTIVE TISSUE (ANATOMY AND PHYSIOLOGY).pdf
CONNECTIVE TISSUE (ANATOMY AND PHYSIOLOGY).pdfDolisha Warbi
 
Unit I herbs as raw materials, biodynamic agriculture.ppt
Unit I herbs as raw materials, biodynamic agriculture.pptUnit I herbs as raw materials, biodynamic agriculture.ppt
Unit I herbs as raw materials, biodynamic agriculture.pptPradnya Wadekar
 
SGK RỐI LOẠN TOAN KIỀM ĐHYHN RẤT HAY VÀ ĐẶC SẮC.pdf
SGK RỐI LOẠN TOAN KIỀM ĐHYHN RẤT HAY VÀ ĐẶC SẮC.pdfSGK RỐI LOẠN TOAN KIỀM ĐHYHN RẤT HAY VÀ ĐẶC SẮC.pdf
SGK RỐI LOẠN TOAN KIỀM ĐHYHN RẤT HAY VÀ ĐẶC SẮC.pdfHongBiThi1
 
SGK RỐI LOẠN KALI MÁU CỰC KỲ QUAN TRỌNG.pdf
SGK RỐI LOẠN KALI MÁU CỰC KỲ QUAN TRỌNG.pdfSGK RỐI LOẠN KALI MÁU CỰC KỲ QUAN TRỌNG.pdf
SGK RỐI LOẠN KALI MÁU CỰC KỲ QUAN TRỌNG.pdfHongBiThi1
 
historyofpsychiatryinindia. Senthil Thirusangu
historyofpsychiatryinindia. Senthil Thirusanguhistoryofpsychiatryinindia. Senthil Thirusangu
historyofpsychiatryinindia. Senthil Thirusangu Medical University
 
AUTONOMIC NERVOUS SYSTEM organization and functions
AUTONOMIC NERVOUS SYSTEM organization and functionsAUTONOMIC NERVOUS SYSTEM organization and functions
AUTONOMIC NERVOUS SYSTEM organization and functionsMedicoseAcademics
 
Using Data Visualization in Public Health Communications
Using Data Visualization in Public Health CommunicationsUsing Data Visualization in Public Health Communications
Using Data Visualization in Public Health Communicationskatiequigley33
 
Clinical Research Informatics Year-in-Review 2024
Clinical Research Informatics Year-in-Review 2024Clinical Research Informatics Year-in-Review 2024
Clinical Research Informatics Year-in-Review 2024Peter Embi
 
pA2 value, Schild plot and pD2 values- applications in pharmacology
pA2 value, Schild plot and pD2 values- applications in pharmacologypA2 value, Schild plot and pD2 values- applications in pharmacology
pA2 value, Schild plot and pD2 values- applications in pharmacologyDeepakDaniel9
 

Dernier (20)

Biologic therapy ice breaking in rheumatology, Case based approach with appli...
Biologic therapy ice breaking in rheumatology, Case based approach with appli...Biologic therapy ice breaking in rheumatology, Case based approach with appli...
Biologic therapy ice breaking in rheumatology, Case based approach with appli...
 
AORTIC DISSECTION and management of aortic dissection
AORTIC DISSECTION and management of aortic dissectionAORTIC DISSECTION and management of aortic dissection
AORTIC DISSECTION and management of aortic dissection
 
FDMA FLAP - The first dorsal metacarpal artery (FDMA) flap is used mainly for...
FDMA FLAP - The first dorsal metacarpal artery (FDMA) flap is used mainly for...FDMA FLAP - The first dorsal metacarpal artery (FDMA) flap is used mainly for...
FDMA FLAP - The first dorsal metacarpal artery (FDMA) flap is used mainly for...
 
SGK LEUKEMIA KINH DÒNG BẠCH CÂU HẠT HAY.pdf
SGK LEUKEMIA KINH DÒNG BẠCH CÂU HẠT HAY.pdfSGK LEUKEMIA KINH DÒNG BẠCH CÂU HẠT HAY.pdf
SGK LEUKEMIA KINH DÒNG BẠCH CÂU HẠT HAY.pdf
 
EXERCISE PERFORMANCE.pptx, Lung function
EXERCISE PERFORMANCE.pptx, Lung functionEXERCISE PERFORMANCE.pptx, Lung function
EXERCISE PERFORMANCE.pptx, Lung function
 
Physiology of Smooth Muscles -Mechanics of contraction and relaxation
Physiology of Smooth Muscles -Mechanics of contraction and relaxationPhysiology of Smooth Muscles -Mechanics of contraction and relaxation
Physiology of Smooth Muscles -Mechanics of contraction and relaxation
 
PAIN/CLASSIFICATION AND MANAGEMENT OF PAIN.pdf
PAIN/CLASSIFICATION AND MANAGEMENT OF PAIN.pdfPAIN/CLASSIFICATION AND MANAGEMENT OF PAIN.pdf
PAIN/CLASSIFICATION AND MANAGEMENT OF PAIN.pdf
 
Trustworthiness of AI based predictions Aachen 2024
Trustworthiness of AI based predictions Aachen 2024Trustworthiness of AI based predictions Aachen 2024
Trustworthiness of AI based predictions Aachen 2024
 
American College of physicians ACP high value care recommendations in rheumat...
American College of physicians ACP high value care recommendations in rheumat...American College of physicians ACP high value care recommendations in rheumat...
American College of physicians ACP high value care recommendations in rheumat...
 
CONNECTIVE TISSUE (ANATOMY AND PHYSIOLOGY).pdf
CONNECTIVE TISSUE (ANATOMY AND PHYSIOLOGY).pdfCONNECTIVE TISSUE (ANATOMY AND PHYSIOLOGY).pdf
CONNECTIVE TISSUE (ANATOMY AND PHYSIOLOGY).pdf
 
Unit I herbs as raw materials, biodynamic agriculture.ppt
Unit I herbs as raw materials, biodynamic agriculture.pptUnit I herbs as raw materials, biodynamic agriculture.ppt
Unit I herbs as raw materials, biodynamic agriculture.ppt
 
SGK RỐI LOẠN TOAN KIỀM ĐHYHN RẤT HAY VÀ ĐẶC SẮC.pdf
SGK RỐI LOẠN TOAN KIỀM ĐHYHN RẤT HAY VÀ ĐẶC SẮC.pdfSGK RỐI LOẠN TOAN KIỀM ĐHYHN RẤT HAY VÀ ĐẶC SẮC.pdf
SGK RỐI LOẠN TOAN KIỀM ĐHYHN RẤT HAY VÀ ĐẶC SẮC.pdf
 
SGK RỐI LOẠN KALI MÁU CỰC KỲ QUAN TRỌNG.pdf
SGK RỐI LOẠN KALI MÁU CỰC KỲ QUAN TRỌNG.pdfSGK RỐI LOẠN KALI MÁU CỰC KỲ QUAN TRỌNG.pdf
SGK RỐI LOẠN KALI MÁU CỰC KỲ QUAN TRỌNG.pdf
 
historyofpsychiatryinindia. Senthil Thirusangu
historyofpsychiatryinindia. Senthil Thirusanguhistoryofpsychiatryinindia. Senthil Thirusangu
historyofpsychiatryinindia. Senthil Thirusangu
 
AUTONOMIC NERVOUS SYSTEM organization and functions
AUTONOMIC NERVOUS SYSTEM organization and functionsAUTONOMIC NERVOUS SYSTEM organization and functions
AUTONOMIC NERVOUS SYSTEM organization and functions
 
Using Data Visualization in Public Health Communications
Using Data Visualization in Public Health CommunicationsUsing Data Visualization in Public Health Communications
Using Data Visualization in Public Health Communications
 
GOUT UPDATE AHMED YEHIA 2024, case based approach with application of the lat...
GOUT UPDATE AHMED YEHIA 2024, case based approach with application of the lat...GOUT UPDATE AHMED YEHIA 2024, case based approach with application of the lat...
GOUT UPDATE AHMED YEHIA 2024, case based approach with application of the lat...
 
Clinical Research Informatics Year-in-Review 2024
Clinical Research Informatics Year-in-Review 2024Clinical Research Informatics Year-in-Review 2024
Clinical Research Informatics Year-in-Review 2024
 
pA2 value, Schild plot and pD2 values- applications in pharmacology
pA2 value, Schild plot and pD2 values- applications in pharmacologypA2 value, Schild plot and pD2 values- applications in pharmacology
pA2 value, Schild plot and pD2 values- applications in pharmacology
 
Rheumatoid arthritis Part 1, case based approach with application of the late...
Rheumatoid arthritis Part 1, case based approach with application of the late...Rheumatoid arthritis Part 1, case based approach with application of the late...
Rheumatoid arthritis Part 1, case based approach with application of the late...
 

comprehensive nursing board exam reviewer

  • 1. Nursing Board Practice Test Compilation FOUNDATION OF PROFESSIONAL NURSING PRACTICE 188 Contents ANSWER KEY - FOUNDATION OF PROFESSIONAL NURSING PRACTICE I: FOUNDATION OF NURSING NURSING PRACTICE.................................................. 199 PRACTICE .......................................................................... 4 COMMUNITY HEALTH NURSING AND CARE OF THE NURSING PRACTICE II ..................................................... 15 MOTHER AND CHILD .................................................... 200 NURSING PRACTICE III .................................................... 26 ANSWER KEY: COMMUNITY HEALTH NURSING AND CARE OF THE MOTHER AND CHILD .......................... 211 NURSING PRACTICE IV.................................................... 36 Comprehensive Exam 1................................................ 213 NURSING PRACTICE V..................................................... 46 CARE OF CLIENTS WITH PHYSIOLOGIC AND TEST I - Foundation of Professional Nursing Practice .... 56 PSYCHOSOCIAL ALTERATIONS...................................... 222 Answers and Rationale – Foundation of Professional ANSWER KEY: CARE OF CLIENTS WITH PHYSIOLOGIC Nursing Practice ......................................................... 66 AND PSYCHOSOCIAL ALTERATIONS ......................... 234 TEST II - Community Health Nursing and Care of the Nursing Practice Test V ................................................ 235 Mother and Child ........................................................... 74 Nursing Practice Test V ................................................ 245 Answers and Rationale – Community Health Nursing and Care of the Mother and Child ............................. 84 TEST I - Foundation of Professional Nursing Practice .. 255 TEST III - Care of Clients with Physiologic and Answers and Rationale – Foundation of Professional Psychosocial Alterations ................................................ 91 Nursing Practice ....................................................... 265 Answers and Rationale – Care of Clients with TEST II - Community Health Nursing and Care of the Physiologic and Psychosocial Alterations ................ 102 Mother and Child ......................................................... 273 TEST IV - Care of Clients with Physiologic and Answers and Rationale – Community Health Nursing Psychosocial Alterations .............................................. 111 and Care of the Mother and Child ........................... 283 Answers and Rationale – Care of Clients with TEST III - Care of Clients with Physiologic and Physiologic and Psychosocial Alterations ................ 122 Psychosocial Alterations .............................................. 290 TEST V - Care of Clients with Physiologic and Psychosocial Answers and Rationale – Care of Clients with Alterations.................................................................... 133 Physiologic and Psychosocial Alterations ................ 301 Answers and Rationale – Care of Clients with TEST IV - Care of Clients with Physiologic and Physiologic and Psychosocial Alterations ................ 144 Psychosocial Alterations .............................................. 310 PART III PRACTICE TEST I FOUNDATION OF NURSING . 153 Answers and Rationale – Care of Clients with Physiologic and Psychosocial Alterations ................ 321 ANSWERS AND RATIONALE – FOUNDATION OF NURSING .................................................................. 158 TEST V - Care of Clients with Physiologic and Psychosocial Alterations.................................................................... 332 PRACTICE TEST II Maternal and Child Health ............... 162 Answers and Rationale – Care of Clients with ANSWERS AND RATIONALE – MATERNAL AND CHILD Physiologic and Psychosocial Alterations ................ 343 HEALTH..................................................................... 167 PART III ......................................................................... 352 MEDICAL SURGICAL NURSING ..................................... 173 PRACTICE TEST I FOUNDATION OF NURSING .............. 352 ANSWERS AND RATIONALE – MEDICAL SURGICAL NURSING .................................................................. 178 ANSWERS AND RATIONALE – FOUNDATION OF NURSING .................................................................. 357 PSYCHIATRIC NURSING ................................................ 180 PRACTICE TEST II Maternal and Child Health ............... 361 ANSWERS AND RATIONALE – PSYCHIATRIC NURSING ................................................................................. 185
  • 2. ANSWERS AND RATIONALE – MATERNAL AND CHILD MEDICAL SURGICAL NURSING Part 1 ........................... 475 HEALTH..................................................................... 366 ANSWERS and RATIONALES for MEDICAL SURGICAL MEDICAL SURGICAL NURSING ..................................... 372 NURSING Part 1 ........................................................ 479 ANSWERS AND RATIONALE – MEDICAL SURGICAL MEDICAL SURGICAL NURSING Part 2 ........................... 481 NURSING .................................................................. 377 MEDICAL SURGICAL NURSING Part 2 ....................... 485 PSYCHIATRIC NURSING ................................................ 379 ANSWERS and RATIONALES for MEDICAL SURGICAL ANSWERS AND RATIONALE – PSYCHIATRIC NURSING NURSING Part 2 ........................................................ 489 ................................................................................. 384 MEDICAL SURGICAL NURSING Part 3 ........................... 491 FUNDAMENTALS OF NURSING PART 1 ........................ 387 ANSWERS and RATIONALES for MEDICAL SURGICAL FUNDAMENTALS OF NURSING PART 2 ........................ 392 NURSING Part 3 ........................................................ 495 ANSWERS and RATIONALES for FUNDAMENTALS OF PSYCHIATRIC NURSING Part 1 ...................................... 497 NURSING PART 2 ...................................................... 397 ANSWERS and RATIONALES for PSYCHIATRIC NURSING FUNDAMENTALS OF NURSING PART 3 ........................ 401 Part 1 ........................................................................ 502 ANSWERS and RATIONALES for FUNDAMENTALS OF PSYCHIATRIC NURSING Part 2 ...................................... 504 NURSING PART 3 ...................................................... 405 ANSWERS and RATIONALES for PSYCHIATRIC NURSING MATERNITY NURSING Part 1 ........................................ 409 Part 2 ........................................................................ 509 ANSWERS and RATIONALES for MATERNITY NURSING PSYCHIATRIC NURSING Part 3 ...................................... 512 Part 1 ........................................................................ 418 ANSWERS and RATIONALES for PSYCHIATRIC NURSING MATERNITY NURSING Part 2 ........................................ 428 Part 3 ........................................................................ 516 Answer for maternity part 2 .................................... 433 PROFESSIONAL ADJUSTMENT ...................................... 519 PEDIATRIC NURSING .................................................... 434 LEADERSHIP and MANAGEMENT ................................. 522 ANSWERS and RATIONALES for PEDIATRIC NURSING NURSING RESEARCH Part 1 .......................................... 532 ................................................................................. 439 NURSING RESEARCH Part 2 .......................................... 542 COMMUNITY HEALTH NURSING Part 1........................ 444 Nursing Research Suggested Answer Key ................ 546 COMMUNITY HEALTH NURSING Part 2........................ 454 2
  • 3. 3
  • 4. 5. Benner’s “Proficient” nurse level is different from the other levels in nursing expertise in the NURSING PRACTICE I: FOUNDATION OF NURSING context of having: PRACTICE a. the ability to organize and plan activities b. having attained an advanced level of SITUATION: Nursing is a profession. The nurse should education have a background on the theories and foundation of c. a holistic understanding and perception nursing as it influenced what is nursing today. of the client d. intuitive and analytic ability in new 1. Nursing is the protection, promotion and situations optimization of health and abilities, prevention of illness and injury, alleviation of suffering SITUATION: The nurse has been asked to administer an through the diagnosis and treatment of human injection via Z TRACK technique. Questions 6 to 10 refer response and advocacy in the care of the to this. individuals, families, communities and the population. This is the most accepted definition 6. The nurse prepares an IM injection for an adult of nursing as defined by the: client using the Z track technique. 4 ml of a. PNA medication is to be administered to the client. b. ANA Which of the following site will you choose? c. Nightingale a. Deltoid d. Henderson b. Rectus femoris c. Ventrogluteal 2. Advancement in Nursing leads to the d. Vastus lateralis development of the Expanded Career Roles. Which of the following is NOT an expanded 7. In infants 1 year old and below, which of the career role for nurses? following is the site of choice for intramuscular a. Nurse practitioner Injection? b. Nurse Researcher a. Deltoid c. Clinical nurse specialist b. Rectus femoris d. Nurse anaesthesiologist c. Ventrogluteal d. Vastus lateralis 3. The Board of Nursing regulated the Nursing profession in the Philippines and is responsible 8. In order to decrease discomfort in Z track for the maintenance of the quality of nursing in administration, which of the following is the country. Powers and duties of the board of applicable? nursing are the following, EXCEPT: a. Pierce the skin quickly and smoothly at a. Issue, suspend, revoke certificates of a 90 degree angle registration b. Inject the medication steadily at around b. Issue subpoena duces tecum, ad 10 minutes per millilitre testificandum c. Pull back the plunger and aspirate for 1 c. Open and close colleges of nursing minute to make sure that the needle did d. Supervise and regulate the practice of not hit a blood vessel nursing d. Pierce the skin slowly and carefully at a 90 degree angle 4. A nursing student or a beginning staff nurse who has not yet experienced enough real situations 9. After injection using the Z track technique, the to make judgments about them is in what stage nurse should know that she needs to wait for a of Nursing Expertise? few seconds before withdrawing the needle and a. Novice this is to allow the medication to disperse into b. Newbie the muscle tissue, thus decreasing the client’s c. Advanced Beginner discomfort. How many seconds should the nurse d. Competent wait before withdrawing the needle? a. 2 seconds 4
  • 5. 5 b. 5 seconds that the patient smokes and drinks coffee. When c. 10 seconds taking the blood pressure of a client who d. 15 seconds recently smoked or drank coffee, how long should the nurse wait before taking the client’s 10. The rationale in using the Z track technique in an blood pressure for accurate reading? intramuscular injection is: a. 15 minutes a. It decreases the leakage of discolouring b. 30 minutes and irritating medication into the c. 1 hour subcutaneous tissues d. 5 minutes b. It will allow a faster absorption of the medication 15. While the client has pulse oximeter on his c. The Z track technique prevent irritation fingertip, you notice that the sunlight is shining of the muscle on the area where the oximeter is. Your action d. It is much more convenient for the nurse will be to: a. Set and turn on the alarm of the SITUATION: A Client was rushed to the emergency room oximeter and you are his attending nurse. You are performing a b. Do nothing since there is no identified vital sign assessment. problem c. Cover the fingertip sensor with a towel 11. All of the following are correct methods in or bedsheet assessment of the blood pressure EXCEPT: d. Change the location of the sensor every a. Take the blood pressure reading on both four hours arms for comparison b. Listen to and identify the phases of 16. The nurse finds it necessary to recheck the blood Korotkoff’s sound pressure reading. In case of such re assessment, c. Pump the cuff to around 50 mmHg the nurse should wait for a period of: above the point where the pulse is a. 15 seconds obliterated b. 1 to 2 minutes d. Observe procedures for infection control c. 30 minutes d. 15 minutes 12. You attached a pulse oximeter to the client. You know that the purpose is to: 17. If the arm is said to be elevated when taking the a. Determine if the client’s hemoglobin blood pressure, it will create a: level is low and if he needs blood a. False high reading transfusion b. False low reading b. Check level of client’s tissue perfusion c. True false reading c. Measure the efficacy of the client’s anti- d. Indeterminate hypertensive medications d. Detect oxygen saturation of arterial 18. You are to assessed the temperature of the blood before symptoms of hypoxemia client the next morning and found out that he develops ate ice cream. How many minutes should you wait before assessing the client’s oral 13. After a few hours in the Emergency Room, The temperature? client is admitted to the ward with an order of a. 10 minutes hourly monitoring of blood pressure. The nurse b. 20 minutes finds that the cuff is too narrow and this will c. 30 minutes cause the blood pressure reading to be: d. 15 minutes a. inconsistent b. low systolic and high diastolic 19. When auscultating the client’s blood pressure c. higher than what the reading should be the nurse hears the following: From 150 mmHg d. lower than what the reading should be to 130 mmHg: Silence, Then: a thumping sound continuing down to 100 mmHg; muffled sound 14. Through the client’s health history, you gather continuing down to 80 mmHg and then silence.
  • 6. What is the client’s blood pressure? to lungs. This can be avoided by: a. 130/80 a. Cleaning teeth and mouth with cotton b. 150/100 swabs soaked with mouthwash to avoid c. 100/80 rinsing the buccal cavity d. 150/100 b. swabbing the inside of the cheeks and lips, tongue and gums with dry cotton 20. In a client with a previous blood pressure of swabs 130/80 4 hours ago, how long will it take to c. use fingers wrapped with wet cotton release the blood pressure cuff to obtain an washcloth to rub inside the cheeks, accurate reading? tongue, lips and ums a. 10-20 seconds d. suctioning as needed while cleaning the b. 30-45 seconds buccal cavity c. 1-1.5 minutes d. 3-3.5 minutes 25. Your client has difficulty of breathing and is mouth breathing most of the time. This causes Situation: Oral care is an important part of hygienic dryness of the mouth with unpleasant odor. Oral practices and promoting client comfort. hygiene is recommended for the client and in addition, you will keep the mouth moistened by 21. An elderly client, 84 years old, is unconscious. using: Assessment of the mouth reveals excessive a. salt solution dryness and presence of sores. Which of the b. petroleum jelly following is BEST to use for oral care? c. water a. lemon glycerine d. mentholated ointment b. Mineral oil c. hydrogen peroxide Situation – Ensuring safety before, during and after a d. Normal saline solution diagnostic procedure is an important responsibility of the nurse. 22. When performing oral care to an unconscious client, which of the following is a special 26. To help Fernan better tolerate the consideration to prevent aspiration of fluids into bronchoscopy, you should instruct him to the lungs? practice which of the following prior to the a. Put the client on a sidelying position procedure? with head of bed lowered a. Clenching his fist every 2 minutes b. Keep the client dry by placing towel b. Breathing in and out through the nose under the chin with his mouth open c. Wash hands and observes appropriate c. Tensing the shoulder muscles while lying infection control on his back d. Clean mouth with oral swabs in a careful d. Holding his breath periodically for 30 and an orderly progression seconds 23. The advantages of oral care for a client include 27. Following a bronchoscopy, which of the all of the following, EXCEPT: following complains to Fernan should be noted a. decreases bacteria in the mouth and as a possible complication: teeth a. Nausea and vomiting b. reduces need to use commercial b. Shortness of breath and laryngeal mouthwash which irritate the buccal stridor mucosa c. Blood tinged sputum and coughing c. improves client’s appearance and self- d. Sore throat and hoarseness confidence d. improves appetite and taste of food 28. Immediately after bronchoscopy, you instructed Fernan to: 24. A possible problem while providing oral care to a. Exercise the neck muscles unconscious clients is the risk of fluid aspiration b. Refrain from coughing and talking 6
  • 7. 7 c. Breathe deeply d. Weber’s test d. Clear his throat 34. A nurse is reviewing the arterial blood gas values 29. Thoracentesis may be performed for cytologic of a client and notes that the ph is 7.31, Pco2 is study of pleural fluid. As a nurse your most 50 mmHg, and the bicarbonate is 27 mEq/L. The important function during the procedure is to: nurse concludes that which acid base a. Keep the sterile equipment from disturbance is present in this client? contamination a. Respiratory acidosis b. Assist the physician b. Metabolic acidosis c. Open and close the three-way stopcock c. Respiratory alkalosis d. Observe the patient’s vital signs d. Metabolic alkalosis 30. Right after thoracentesis, which of the following 35. Allen’s test checks the patency of the: is most appropriate intervention? a. Ulnar artery a. Instruct the patient not to cough or deep b. Carotid artery breathe for two hours c. Radial artery b. Observe for symptoms of tightness of d. Brachial artery chest or bleeding c. Place an ice pack to the puncture site Situation 6: Eileen, 45 years old is admitted to the d. Remove the dressing to check for hospital with a diagnosis of renal calculi. She is bleeding experiencing severe flank pain, nauseated and with a temperature of 39 0C. Situation: Knowledge of the acid-base disturbance and the functions of the electrolytes is necessary to 36. Given the above assessment data, the most determine appropriate intervention and nursing actions. immediate goal of the nurse would be which of the following? 31. A client with diabetes milletus has a blood a. Prevent urinary complication glucose level of 644 mg/dL. The nurse interprets b. maintains fluid and electrolytes that this client is at most risk for the c. Alleviate pain development of which type of acid-base d. Alleviating nausea imbalance? a. Respiratory acidosis 37. After IVP a renal stone was confirmed, a left b. Respiratory alkalosis nephrectomy was done. Her post-operative c. Metabolic acidosis order includes “daily urine specimen to be sent d. Metabolic alkalosis to the laboratory”. Eileen has a foley catheter attached to a urinary drainage system. How will 32. In a client in the health care clinic, arterial blood you collect the urine specimen? gas analysis gives the following results: pH 7.48, a. remove urine from drainage tube with PCO2 32 mmHg, PO2 94 mmHg, HCO3 24 mEq/L. sterile needle and syringe and empty The nurse interprets that the client has which urine from the syringe into the acid base disturbance? specimen container a. Respiratory acidosis b. empty a sample urine from the b. Metabolic acidosis collecting bag into the specimen c. Respiratory alkalosis container d. Metabolic alkalosis c. Disconnect the drainage tube from the indwelling catheter and allow urine to 33. A client has an order for ABG analysis on radial flow from catheter into the specimen artery specimens. The nurse ensures that which container. of the following has been performed or tested d. Disconnect the drainage from the before the ABG specimens are drawn? collecting bag and allow the urine to a. Guthrie test flow from the catheter into the b. Romberg’s test specimen container. c. Allen’s test
  • 8. 38. Where would the nurse tape Eileen’s indwelling regulation is secreted in the: catheter in order to reduce urethral irritation? a. Thyroid gland a. to the patient’s inner thigh b. Parathyroid gland b. to the patient’ buttocks c. Hypothalamus c. to the patient’s lower thigh d. Anterior pituitary gland d. to the patient lower abdomen 45. While Parathormone, a hormone that negates 39. Which of the following menu is appropriate for the effect of calcitonin is secreted by the: one with low sodium diet? a. Thyroid gland a. instant noodles, fresh fruits and ice tea b. Parathyroid gland b. ham and cheese sandwich, fresh fruits c. Hypothalamus and vegetables d. Anterior pituitary gland c. white chicken sandwich, vegetable salad and tea Situation: The staff nurse supervisor requests all the staff d. canned soup, potato salad, and diet soda nurses to “brainstorm” and learn ways to instruct diabetic clients on self-administration of insulin. She 40. How will you prevent ascending infection to wants to ensure that there are nurses available daily to Eileen who has an indwelling catheter? do health education classes. a. see to it that the drainage tubing touches the level of the urine 46. The plan of the nurse supervisor is an example of b. change he catheter every eight hours a. in service education process c. see to it that the drainage tubing does b. efficient management of human not touch the level of the urine resources d. clean catheter may be used since c. increasing human resources urethral meatus is not a sterile area d. primary prevention Situation: Hormones are secreted by the various glands 47. When Mrs. Guevarra, a nurse, delegates aspects in the body. Basic knowledge of the endocrine system is of the clients care to the nurse-aide who is an necessary. unlicensed staff, Mrs. Guevarra a. makes the assignment to teach the staff 41. Somatocrinin or the Growth hormone releasing member hormone is secreted by the: b. is assigning the responsibility to the a. Hypothalamus aide but not the accountability for b. Posterior pituitary gland those tasks c. Anterior pituitary gland c. does not have to supervise or evaluate d. Thyroid gland the aide d. most know how to perform task 42. All of the following are secreted by the anterior delegated pituitary gland except: a. Somatotropin/Growth hormone 48. Connie, the new nurse, appears tired and b. Thyroid stimulating hormone sluggish and lacks the enthusiasm she had six c. Follicle stimulating hormone weeks ago when she started the job. The nurse d. Gonadotropin hormone releasing supervisor should hormone a. empathize with the nurse and listen to her 43. All of the following hormones are hormones b. tell her to take the day off secreted by the Posterior pituitary gland except: c. discuss how she is adjusting to her new a. Vasopressin job b. Anti-diuretic hormone d. ask about her family life c. Oxytocin d. Growth hormone 49. Process of formal negotiations of working conditions between a group of registered nurses 44. Calcitonin, a hormone necessary for calcium and employer is 8
  • 9. 9 a. grievance d. It should disclose previous diagnosis, b. arbitration prognosis and alternative treatments c. collective bargaining available for the client d. strike 55. Delegation is the process of assigning tasks that 50. You are attending a certification on can be performed by a subordinate. The RN cardiopulmonary resuscitation (CPR) offered and should always be accountable and should not required by the hospital employing you. This is lose his accountability. Which of the following is a. professional course towards credits a role included in delegation? b. in-service education a. The RN must supervise all delegated c. advance training tasks d. continuing education b. After a task has been delegated, it is no longer a responsibility of the RN Situation: As a nurse, you are aware that proper c. The RN is responsible and accountable documentation in the patient chart is your responsibility. for the delegated task in adjunct with the delegate 51. Which of the following is not a legally binding d. Follow up with a delegated task is document but nevertheless very important in necessary only if the assistive personnel the care of all patients in any health care is not trustworthy setting? a. Bill of rights as provided in the Philippine Situation: When creating your lesson plan for constitution cerebrovascular disease or STROKE. It is important to b. Scope of nursing practice as defined by include the risk factors of stroke. RA 9173 c. Board of nursing resolution adopting the 56. The most important risk factor is: code of ethics a. Cigarette smoking d. Patient’s bill of rights b. binge drinking c. Hypertension 52. A nurse gives a wrong medication to the client. d. heredity Another nurse employed by the same hospital as a risk manager will expect to receive which of 57. Part of your lesson plan is to talk about etiology the following communication? or cause of stroke. The types of stroke based on a. Incident report cause are the following EXCEPT: b. Nursing kardex a. Embolic stroke c. Oral report b. diabetic stroke d. Complain report c. Hemorrhagic stroke d. thrombotic stroke 53. Performing a procedure on a client in the absence of an informed consent can lead to 58. Hemmorhagic stroke occurs suddenly usually which of the following charges? when the person is active. All are causes of a. Fraud hemorrhage, EXCEPT: b. Harassment a. phlebitis c. Assault and battery b. damage to blood vessel d. Breach of confidentiality c. trauma d. aneurysm 54. Which of the following is the essence of informed consent? 59. The nurse emphasizes that intravenous drug a. It should have a durable power of abuse carries a high risk of stroke. Which drug is attorney closely linked to this? b. It should have coverage from an a. Amphetamines insurance company b. shabu c. It should respect the client’s freedom c. Cocaine from coercion d. Demerol
  • 10. d. Iron 75 mg/100 ml 60. A participant in the STROKE class asks what is a risk factor of stroke. Your best response is: 65. Which of the following laboratory test result a. “More red blood cells thicken blood indicate presence of an infectious process? and make clots more possible.” a. Erythrocyte sedimentation rate (ESR) 12 b. “Increased RBC count is linked to high mm/hr cholesterol.” b. White blood cells (WBC) 18,000/mm3 c. “More red blood cell increases c. Iron 90 g/100ml hemoglobin content.” d. Neutrophils 67% d. “High RBC count increases blood pressure.” Situation: Pleural effusion is the accumulation of fluid in the pleural space. Questions 66 to 70 refer to this. Situation: Recognition of normal values is vital in assessment of clients with various disorders. 66. Which of the following is a finding that the nurse will be able to assess in a client with Pleural 61. A nurse is reviewing the laboratory test results effusion? for a client with a diagnosis of severe a. Reduced or absent breath sound at the dehydration. The nurse would expect the base of the lungs, dyspnea, tachpynea hematocrit level for this client to be which of the and shortness of breath following? b. Hypoxemia, hypercapnea and a. 60% respiratory acidosis b. 47% c. Noisy respiration, crackles, stridor and c. 45% wheezing d. 32% d. Tracheal deviation towards the affected side, increased fremitus and loud breath 62. A nurse is reviewing the electrolyte results of an sounds assigned client and notes that the potassium level is 5.6 mEq/L. Which of the following would 67. Thoracentesis is performed to the client with the nurse expect to note on the ECG as a result effusion. The nurse knows that the removal of of this laboratory value? fluid should be slow. Rapid removal of fluid in a. ST depression thoracentesis might cause: b. Prominent U wave a. Pneumothorax c. Inverted T wave b. Cardiovascular collapse d. Tall peaked T waves c. Pleurisy or Pleuritis d. Hypertension 63. A nurse is reviewing the electrolyte results of an assigned client and notes that the potassium 68. 3 Days after thoracentesis, the client again level is 3.2 mEq/L. Which of the following would exhibited respiratory distress. The nurse will the nurse expect to note on the ECG as a result know that pleural effusion has reoccurred when of this laboratory value? she noticed a sharp stabbing pain during a. U waves inspiration. The physician ordered a closed tube b. Elevated T waves thoracotomy for the client. The nurse knows c. Absent P waves that the primary function of the chest tube is to: d. Elevated ST Segment a. Restore positive intrathoracic pressure b. Restore negative intrathoracic pressure 64. Dorothy underwent diagnostic test and the c. To visualize the intrathoracic content result of the blood examination are back. On d. As a method of air administration via reviewing the result the nurse notices which of ventilator the following as abnormal finding? a. Neutrophils 60% 69. The chest tube is functioning properly if: b. White blood cells (WBC) 9000/mm a. There is an oscillation c. Erythrocyte sedimentation rate (ESR) is b. There is no bubbling in the drainage 39 mm/hr bottle 10
  • 11. 11 c. There is a continuous bubbling in the waterseal 75. This form of Health Insurance provides d. The suction control bottle has a comprehensive prepaid health services to continuous bubbling enrollees for a fixed periodic payment. a. Health Maintenance Organization 70. In a client with pleural effusion, the nurse is b. Medicare instructing appropriate breathing technique. c. Philippine Health Insurance Act Which of the following is included in the d. Hospital Maintenance Organization teaching? a. Breath normally Situation: Nursing ethics is an important part of the b. Hold the breath after each inspiration nursing profession. As the ethical situation arises, so is for 1 full minute the need to have an accurate and ethical decision c. Practice abdominal breathing making. d. Inhale slowly and hold the breath for 3 to 5 seconds after each inhalation 76. The purpose of having a nurses’ code of ethics is: a. Delineate the scope and areas of nursing SITUATION: Health care delivery system affects the practice health status of every filipino. As a Nurse, Knowledge of b. identify nursing action recommended for this system is expected to ensure quality of life. specific health care situations c. To help the public understand 71. When should rehabilitation commence? professional conduct expected of a. The day before discharge nurses b. When the patient desires d. To define the roles and functions of the c. Upon admission health care givers, nurses, clients d. 24 hours after discharge 77. The principles that govern right and proper 72. What exemplified the preventive and promotive conduct of a person regarding life, biology and programs in the hospital? the health professionals is referred to as: a. Hospital as a center to prevent and a. Morality control infection b. Religion b. Program for smokers c. Values c. Program for alcoholics and drug addicts d. Bioethics d. Hospital Wellness Center 78. A subjective feeling about what is right or wrong 73. Which makes nursing dynamic? is said to be: a. Every patient is a unique physical, a. Morality emotional, social and spiritual being b. Religion b. The patient participate in the overall c. Values nursing care plan d. Bioethics c. Nursing practice is expanding in the light of modern developments that takes 79. Values are said to be the enduring believe about place a worth of a person, ideas and belief. If Values d. The health status of the patient is are going to be a part of a research, this is constantly changing and the nurse must categorized under: be cognizant and responsive to these a. Qualitative changes b. Experimental c. Quantitative 74. Prevention is an important responsibility of the d. Non Experimental nurse in: a. Hospitals 80. The most important nursing responsibility where b. Community ethical situations emerge in patient care is to: c. Workplace a. Act only when advised that the action is d. All of the above ethically sound
  • 12. b. Not takes sides, remain neutral and fair c. Assume that ethical questions are the 85. Based on the Code of Ethics for Filipino Nurses, responsibility of the health team what is regarded as the hallmark of nursing d. Be accountable for his or her own responsibility and accountability? actions a. Human rights of clients, regardless of creed and gender 81. Why is there an ethical dilemma? b. The privilege of being a registered a. the choices involved do not appear to be professional nurse clearly right or wrong c. Health, being a fundamental right of b. a client’s legal right co-exist with the every individual nurse’s professional obligation d. Accurate documentation of actions and c. decisions has to be made based on outcomes societal norms. d. decisions has to be mad quickly, often Situation: As a profession, nursing is dynamic and its under stressful conditions practice is directed by various theoretical models. To demonstrate caring behaviour, the nurse applies various 82. According to the code of ethics, which of the nursing models in providing quality nursing care. following is the primary responsibility of the nurse? 86. When you clean the bedside unit and regularly a. Assist towards peaceful death attend to the personal hygiene of the patient as b. Health is a fundamental right well as in washing your hands before and after a c. Promotion of health, prevention of procedure and in between patients, you indent illness, alleviation of suffering and to facilitate the body’s reparative processes. restoration of health Which of the following nursing theory are you d. Preservation of health at all cost applying in the above nursing action? a. Hildegard Peplau 83. Which of the following is TRUE about the Code b. Dorothea Orem of Ethics of Filipino Nurses, except: c. Virginia Henderson a. The Philippine Nurses Association for d. Florence Nightingale being the accredited professional organization was given the privilege to 87. A communication skill is one of the important formulate a Code of Ethics for Nurses competencies expected of a nurse. Interpersonal which the Board of Nursing process is viewed as human to human promulgated relationship. This statement is an application of b. Code for Nurses was first formulated in whose nursing model? 1982 published in the Proceedings of the a. Joyce Travelbee Third Annual Convention of the PNA b. Martha Rogers House of Delegates c. Callista Roy c. The present code utilized the Code of d. Imogene King Good Governance for the Professions in the Philippines 88. The statement “the health status of an individual d. Certificates of Registration of registered is constantly changing and the nurse must be nurses may be revoked or suspended for cognizant and responsive to these changes” best violations of any provisions of the Code explains which of the following facts about of Ethics. nursing? a. Dynamic 84. Violation of the code of ethics might equate to b. Client centred the revocation of the nursing license. Who c. Holistic revokes the license? d. Art a. PRC b. PNA 89. Virginia Henderson professes that the goal of c. DOH nursing is to work interdependently with other d. BON health care working in assisting the patient to 12
  • 13. 13 gain independence as quickly as possible. Which include: of the following nursing actions best a. Prescription of the doctor to the demonstrates this theory in taking care of a 94 patient’s illness year old client with dementia who is totally b. Plan of care for patient immobile? c. Patient’s perception of one’s illness a. Feeds the patient, brushes his teeth, d. Nursing problem and Nursing diagnosis gives the sponge bath b. Supervise the watcher in rendering 94. The medical records that are organized into patient his morning care separate section from doctors or nurses has c. Put the patient in semi fowler’s position, more disadvantages than advantages. This is set the over bed table so the patient can classified as what type of recording? eat by himself, brush his teeth and a. POMR sponge himself b. Modified POMR d. Assist the patient to turn to his sides and c. SOAPIE allow him to brush and feed himself only d. SOMR when he feels ready 95. Which of the following is the advantage of SOMR 90. In the self-care deficit theory by Dorothea Orem, or Traditional recording? nursing care becomes necessary when a patient a. Increases efficiency in data gathering is unable to fulfil his physiological, psychological b. Reinforces the use of the nursing and social needs. A pregnant client needing process prenatal check-up is classified as: c. The caregiver can easily locate proper a. Wholly compensatory section for making charting entries b. Supportive Educative d. Enhances effective communication c. Partially compensatory among health care team members d. Non compensatory Situation: June is a 24 year old client with symptoms of Situation: Documentation and reporting are just as dyspnea, absent breath sounds on the right lung and important as providing patient care, As such, the nurse chest x ray revealed pleural effusion. The physician will must be factual and accurate to ensure quality perform thoracentesis. documentation and reporting. 96. Thoracentesis is useful in treating all of the 91. Health care reports have different purposes. The following pulmonary disorders except: availability of patients’ record to all health team a. Hemothorax members demonstrates which of the following b. Hydrothorax purposes: c. Tuberculosis a. Legal documentation d. Empyema b. Research c. Education 97. Which of the following psychological preparation d. Vehicle for communication is not relevant for him? a. Telling him that the gauge of the needle 92. When a nurse commits medication error, she and anesthesia to be used should accurately document client’s response b. Telling him to keep still during the and her corresponding action. This is very procedure to facilitate the insertion of important for which of the following purposes: the needle in the correct place a. Research c. Allow June to express his feelings and b. Legal documentation concerns c. Nursing Audit d. Physician’s explanation on the purpose d. Vehicle for communication of the procedure and how it will be done 93. POMR has been widely used in many teaching 98. Before thoracentesis, the legal consideration you hospitals. One of its unique features is SOAPIE must check is: charting. The P in SOAPIE charting should a. Consent is signed by the client
  • 14. b. Medicine preparation is correct c. Position of the client is correct d. Consent is signed by relative and physician 99. As a nurse, you know that the position for June before thoracentesis is: a. Orthopneic b. Low fowlers c. Knee-chest d. Sidelying position on the affected side 100. Which of the following anaesthetics drug is used for thoracentesis? a. Procaine 2% b. Demerol 75 mg c. Valium 250 mg d. Phenobartbital 50 mg 14
  • 15. 15 D. Follicle stimulating hormone NURSING PRACTICE II 5. The following month, Mariah suspects she is pregnant. Her urine is positive for Human Situation: Mariah is a 31 year old lawyer who has been chorionic gonadotrophin. Which structure married for 6 months. She consults you for guidance in produces Hcg? relation with her menstrual cycle and her desire to get A. Pituitary gland pregnant. B. Trophoblastic cells of the embryo C. Uterine deciduas 1. She wants to know the length of her menstrual D. Ovarian follicles cycle. Her previous menstrual period is October 22 to 26. Her LMB is November 21. Which of the Situation: Mariah came back and she is now pregnant. following number of days will be your correct response? 6. At 5 month gestation, which of the following A. 29 fetal development would probably be achieve? B. 28 A. Fetal movement are felt by Mariah C. 30 B. Vernix caseosa covers the entire body D. 31 C. Viable if delivered within this period D. Braxton hicks contractions are observed 2. You advised her to observe and record the signs of Ovulation. Which of the following signs will 7. The nurse palpates the abdomen of Mariah. she likely note down? Now At 5 month gestation, What level of the 1. A 1 degree Fahrenheit rise in basal body abdomen can the fundic height be palpated? temperature A. Symphysis pubis 2. Cervical mucus becomes copious and B. Midpoint between the umbilicus and the clear xiphoid process 3. One pound increase in weight C. Midpoint between the symphysis pubis 4. Mittelschmerz and the umbilicus A. 1, 2, 4 D. Umbilicus B. 1, 2, 3 C. 2, 3, 4 8. She worries about her small breasts, thinking D. 1, 3, 4 that she probably will not be able to breastfeed her baby. Which of the following responses of 3. You instruct Mariah to keep record of her basal the nurse is correct? temperature every day, which of the following A. “The size of your breast will not affect instructions is incorrect? your lactation” A. If coitus has occurred; this should be B. “You can switch to bottle feeding” reflected in the chart C. “You can try to have exercise to increase B. It is best to have coitus on the evening the size of your breast” following a drop in BBT to become D. “Manual expression of milk is possible” pregnant C. Temperature should be taken 9. She tells the nurse that she does not take milk immediately after waking and before regularly. She claims that she does not want to getting out of bed gain too much weight during her pregnancy. D. BBT is lowest during the secretory Which of the following nursing diagnosis is a phase priority? A. Potential self-esteem disturbance 4. She reports an increase in BBT on December 16. related to physiologic changes in Which hormone brings about this change in her pregnancy BBT? B. Ineffective individual coping related to A. Estrogen physiologic changes in pregnancy B. Gonadotropine C. Fear related to the effects of pregnancy C. Progesterone D. Knowledge deficit regarding nutritional
  • 16. requirements of pregnancies related to 15. While talking with Susan, 2 new patients arrived lack of information sources and they are covered with large towels and the nurse noticed that there are many cameraman 10. Which of the following interventions will likely and news people outside of the OPD. Upon ensure compliance of Mariah? assessment the nurse noticed that both of them A. Incorporate her food preferences that are still nude and the male client’s penis is still are adequately nutritious in her meal inside the female client’s vagina and the male plan client said that “I can’t pull it”. Vaginismus was B. Consistently counsel toward optimum your first impression. You know that The nutritional intake psychological cause of Vaginismus is related to: C. Respect her right to reject dietary A. The male client inserted the penis too information if she chooses deeply that it stimulates vaginal closure D. Inform her of the adverse effects of B. The penis was too large that is why the inadequate nutrition to her fetus vagina triggered its defense to attempt to close it Situation: Susan is a patient in the clinic where you work. C. The vagina does not want to be She is inquiring about pregnancy. penetrated D. It is due to learning patterns of the 11. Susan tells you she is worried because she female client where she views sex as develops breasts later than most of her friends. bad or sinful Breast development is termed as: A. Adrenarche Situation: Overpopulation is one problem in the B. Thelarche Philippines that causes economic drain. Most Filipinos C. Mamarche are against in legalizing abortion. As a nurse, Mastery of D. Menarche contraception is needed to contribute to the society and economic growth. 12. Kevin, Susan’s husband tells you that he is considering vasectomy After the birth of their 16. Supposed that Dana, 17 years old, tells you she new child. Vasectomy involves the incision of wants to use fertility awareness method of which organ? contraception. How will she determine her A. The testes fertile days? B. The epididymis A. She will notice that she feels hot, as if C. The vas deferens she has an elevated temperature. D. The scrotum B. She should assess whether her cervical mucus is thin, copious, clear and 13. On examination, Susan has been found of having watery. a cystocele. A cystocele is: C. She should monitor her emotions for A. A sebaceous cyst arising from the vulvar sudden anger or crying fold D. She should assess whether her breasts B. Protrusion of intestines into the vagina feel sensitive to cool air C. Prolapse of the uterus into the vagina D. Herniation of the bladder into the 17. Dana chooses to use COC as her family planning vaginal wall method. What is the danger sign of COC you would ask her to report? 14. Susan typically has menstrual cycle of 34 days. A. A stuffy or runny nose She told you she had coitus on days 8, 10, 15 and B. Slight weight gain 20 of her menstrual cycle. Which is the day on C. Arthritis like symptoms which she is most likely to conceive? D. Migraine headache A. 8th day B. Day 15 18. Dana asks about subcutaneous implants and she C. 10th day asks, how long will these implants be effective. D. Day 20 Your best answer is: A. One month 16
  • 17. 17 B. Five years C. Twelve months 23. Another client named Lilia is diagnosed as having D. 10 years endometriosis. This condition interferes with fertility because: 19. Dana asks about female condoms. Which of the A. Endometrial implants can block the following is true with regards to female fallopian tubes condoms? B. The uterine cervix becomes inflamed A. The hormone the condom releases and swollen might cause mild weight gain C. The ovaries stop producing adequate B. She should insert the condom before estrogen any penile penetration D. Pressure on the pituitary leads to C. She should coat the condom with decreased FSH levels spermicide before use D. Female condoms, unlike male condoms, 24. Lilia is scheduled to have a are reusable hysterosalphingogram. Which of the following instructions would you give her regarding this 20. Dana has asked about GIFT procedure. What procedure? makes her a good candidate for GIFT? A. She will not be able to conceive for 3 A. She has patent fallopian tubes, so months after the procedure fertilized ova can be implanted on them B. The sonogram of the uterus will reveal B. She is RH negative, a necessary any tumors present stipulation to rule out RH incompatibility C. Many women experience mild bleeding C. She has normal uterus, so the sperm can as an after effect be injected through the cervix into it D. She may feel some cramping when the D. Her husband is taking sildenafil, so all dye is inserted sperms will be motile 25. Lilia’s cousin on the other hand, knowing nurse Situation: Nurse Lorena is a Family Planning and Lorena’s specialization asks what artificial Infertility Nurse Specialist and currently attends to insemination by donor entails. Which would be FAMILY PLANNING CLIENTS AND INFERTILE COUPLES. your best answer if you were Nurse Lorena? The following conditions pertain to meeting the nursing A. Donor sperm are introduced vaginally needs of this particular population group. into the uterus or cervix B. Donor sperm are injected intra- 21. Dina, 17 years old, asks you how a tubal ligation abdominally into each ovary prevents pregnancy. Which would be the best C. Artificial sperm are injected vaginally to answer? test tubal patency A. Prostaglandins released from the cut D. The husband’s sperm is administered fallopian tubes can kill sperm intravenously weekly B. Sperm cannot enter the uterus because the cervical entrance is blocked. Situation: You are assigned to take care of a group of C. Sperm can no longer reach the ova, patients across the lifespan. because the fallopian tubes are blocked D. The ovary no longer releases ova as 26. Pain in the elder persons requires careful there is nowhere for them to go. assessment because they: A. experienced reduce sensory perception 22. The Dators are a couple undergoing testing for B. have increased sensory perception infertility. Infertility is said to exist when: C. are expected to experience chronic pain A. A woman has no uterus D. have a decreased pain threshold B. A woman has no children C. A couple has been trying to conceive for 27. Administration of analgesics to the older persons 1 year requires careful patient assessment because D. A couple has wanted a child for 6 older people: months A. are more sensitive to drugs
  • 18. B. have increased hepatic, renal and D. Chronic poverty gastrointestinal function C. have increased sensory perception 34. Which of the following signs and symptoms D. mobilize drugs more rapidly would you most likely find when assessing and infant with Arnold-Chiari malformation? 28. The elderly patient is at higher risk for urinary A. Weakness of the leg muscles, loss of incontinence because of: sensation in the legs, and restlessness A. increased glomerular filtration B. Difficulty swallowing, diminished or B. decreased bladder capacity absent gag reflex, and respiratory C. diuretic use distress D. dilated urethra C. Difficulty sleeping, hypervigilant, and an arching of the back 29. Which of the following is the MOST COMMON D. Paradoxical irritability, diarrhea, and sign of infection among the elderly? vomiting. A. decreased breath sounds with crackles B. pain 35. A parent calls you and frantically reports that her C. fever child has gotten into her famous ferrous sulfate D. change in mental status pills and ingested a number of these pills. Her child is now vomiting, has bloody diarrhea, and is 30. Priorities when caring for the elderly trauma complaining of abdominal pain. You will tell the patient: mother to: A. circulation, airway, breathing A. Call emergency medical services (EMS) B. airway, breathing, disability (neurologic) and get the child to the emergency room C. disability (neurologic), airway, breathing B. Relax because these symptoms will pass D. airway, breathing, circulation and the child will be fine C. Administer syrup of ipecac 31. Preschoolers are able to see things from which D. Call the poison control center of the following perspectives? A. Their peers 36. A client says she heard from a friend that you B. Their own and their mother’s stop having periods once you are on the “pill”. C. Their own and their caregivers’ The most appropriate response would be: D. Only their own A. “The pill prevents the uterus from making such endometrial lining, that is 32. In conflict management, the win-win approach why periods may often be scant or occurs when: skipped occasionally.” A. There are two conflicts and the parties B. “If your friend has missed her period, agree to each one she should stop taking the pills and get a B. Each party gives in on 50% of the pregnancy test as soon as possible.” disagreements making up the conflict C. “The pill should cause a normal C. Both parties involved are committed to menstrual period every month. It solving the conflict sounds like your friend has not been D. The conflict is settled out of court so the taking the pills properly.” legal system and the parties win D. “Missed period can be very dangerous and may lead to the formation of 33. According to the social-interactional perspective precancerous cells.” of child abuse and neglect, four factors place the family members at risk for abuse. These risk 37. The nurse assessing newborn babies and infants factors are the family members at risk for abuse. during their hospital stay after birth will notice These risk factors are the family itself, the which of the following symptoms as a primary caregiver, the child, and manifestation of Hirschsprung’s disease? A. The presence of a family crisis A. A fine rash over the trunk B. The national emphasis on sex B. Failure to pass meconium during the C. Genetics first 24 to 48 hours after birth 18
  • 19. 19 C. The skin turns yellow and then brown release over the first 48 hours of life B. a woman is less able to keep the D. High-grade fever laceration clean because of her fatigue C. healing is limited during pregnancy so 38. A client is 7 months pregnant and has just been these will not heal until after birth diagnosed as having a partial placenta previa. D. increased bleeding can occur from She is stable and has minimal spotting and is uterine pressure on leg veins being sent home. Which of these instructions to the client may indicate a need for further 43. In working with the caregivers of a client with an teaching? acute or chronic illness, the nurse would: A. Maintain bed rest with bathroom A. Teach care daily and let the caregivers privileges do a return demonstration just before B. Avoid intercourse for three days. discharge C. Call if contractions occur. B. Difficulty swallowing, diminished or D. Stay on left side as much as possible absent gag reflex, and respiratory when lying down. distress. C. Difficulty sleeping, hypervigilant, and an 39. A woman has been rushed to the hospital with arching of the back ruptured membrane. Which of the following D. Paradoxical irritability, diarrhea, and should the nurse check first? vomiting A. Check for the presence of infection B. Assess for Prolapse of the umbilical 44. Which of the following roles BEST exemplifies cord the expanded role of the nurse? C. Check the maternal heart rate A. Circulating nurse in surgery D. Assess the color of the amniotic fluid B. Medication nurse C. Obstetrical nurse 40. The nurse notes that the infant is wearing a D. Pediatric nurse practitioner plastic-coated diaper. If a topical medication were to be prescribed and it were to go on the 45. According to DeRosa and Kochura’s (2006) stomachs or buttocks, the nurse would teach the article entitled “Implement Culturally Competent caregivers to: Health Care in your work place,” cultures have A. avoid covering the area of the topical different patterns of verbal and nonverbal medication with the diaper communication. Which difference does? B. avoid the use of clothing on top of the A. NOT necessarily belong? diaper B. Personal behavior C. put the diaper on as usual C. Subject matter D. apply an icepack for 5 minutes to the D. Eye contact outside of the diaper E. Conversational style 41. Which of the following factors is most important 46. You are the nurse assigned to work with a child in determining the success of relationships used with acute glomerulonephritis. By following the in delivering nursing care? prescribed treatment regimen, the child A. Type of illness of the client experiences a remission. You are now checking B. Transference and counter transference to make sure the child does not have a relapse. C. Effective communication Which finding would most lead you to the D. Personality of the participants conclusion that a relapse is happening? A. Elevated temperature, cough, sore 42. Grace sustained a laceration on her leg from throat, changing complete blood count automobile accident. Why are lacerations of (CBC) with diiferential lower extremities potentially more serious B. A urine dipstick measurement of 2+ among pregnant women than other? proteinuria or more for 3 days, or the A. lacerations can provoke allergic child found to have 3-4+ proteinutria responses due to gonadotropic hormone plus edema.
  • 20. C. The urine dipstick showing glucose in the urine for 3 days, extreme thirst, increase 51. If a child with diarrhea registers two signs in the in urine output, and a moon face. yellow row in the IMCI chart, we can classify the D. A temperature of 37.8 degrees (100 patient as: degrees F), flank pain, burning A. Moderate dehydration frequency, urgency on voiding, and B. Severe dehydration cloudy urine. C. Some dehydration D. No dehydration 47. The nurse is working with an adolescent who complains of being lonely and having a lack of 52. Celeste has had diarrhea for 8 days. There is no fulfillment in her life. This adolescent shies away blood in the stool, he is irritable, his eyes are from intimate relationships at times yet at other sunken, the nurse offers fluid to Celeste and he times she appears promiscuous. The nurse will drinks eagerly. When the nurse pinched the likely work with this adolescent in which of the abdomen it goes back slowly. How will you following areas? classify Celeste’s illness? A. Isolation A. Moderate dehydration B. Lack of fulfillment B. Severe dehydration C. Loneliness C. Some dehydration D. Identity D. No dehydration 48. The use of interpersonal decision making, 53. A child who is 7 weeks has had diarrhea for 14 psychomotor skills, and application of days but has no sign of dehydration is classified knowledge expected in the role of a licensed as: health care professional in the context of public A. Persistent diarrhea health welfare and safety is an example of: B. Dysentery A. Delegation C. Severe dysentery B. Responsibility D. Severe persistent diarrhea C. Supervision D. Competence 54. The child with no dehydration needs home treatment. Which of the following is not 49. The painful phenomenon known as “back labor” included in the rules for home treatment in this occurs in a client whose fetus in what position? case? A. Brow position A. Forced fluids B. Breech position B. When to return C. Right Occipito-Anterior Position C. Give vitamin A supplement D. Left Occipito-Posterior Position D. Feeding more 50. FOCUS methodology stands for: 55. Fever as used in IMCI includes: A. Focus, Organize, Clarify, Understand A. Axillary temperature of 37.5 or higher and Solution B. Rectal temperature of 38 or higher B. Focus, Opportunity, Continuous, Utilize, C. Feeling hot to touch Substantiate D. All of the above C. Focus, Organize, Clarify, Understand, E. A and C only Substantiate D. Focus, Opportunity, Continuous Situation: Prevention of Dengue is an important nursing (process), Understand, Solution responsibility and controlling it’s spread is a priority once outbreak has been observed. SITUATION: The infant and child mortality rate in the low to middle income countries is ten times higher than 56. An important role of the community health industrialized countries. In response to this, the WHO nurse in the prevention and control of Dengue and UNICEF launched the protocol Integrated H-fever includes: Management of Childhood Illnesses to reduce the A. Advising the elimination of vectors by morbidity and mortality against childhood illnesses. keeping water containers covered 20
  • 21. 21 B. Conducting strong health education health worker should first: drives/campaign directed towards A. Identify the myths and misconceptions proper garbage disposal prevailing in the community C. Explaining to the individuals, families, B. Identify the source of these myths and groups and community the nature of misconceptions the disease and its causation C. Explain how and why these myths came D. Practicing residual spraying with about insecticides D. Select the appropriate IEC strategies to correct them 57. Community health nurses should be alert in observing a Dengue suspect. The following is 62. How many percent of measles are prevented by NOT an indicator for hospitalization of H-fever immunization at 9 months of age? suspects? A. 80% A. Marked anorexia, abdominal pain and B. 99% vomiting C. 90% B. Increasing hematocrit count D. 95% C. Cough of 30 days D. Persistent headache 63. After TT3 vaccination a mother is said to be protected to tetanus by around: 58. The community health nurses’ primary concern A. 80% in the immediate control of hemorrhage among B. 99% patients with dengue is: C. 85% A. Advising low fiber and non-fat diet D. 90% B. Providing warmth through light weight covers 64. If ever convulsions occur after administering C. Observing closely the patient for vital DPT, what should the nurse best suggest to the signs leading to shock mother? D. Keeping the patient at rest A. Do not continue DPT vaccination anymore 59. Which of these signs may NOT be REGARDED as B. Advise mother to comeback after 1 week a truly positive signs indicative of Dengue H- C. Give DT instead of DPT fever? D. Give pertussis of the DPT and remove DT A. Prolonged bleeding time B. Appearance of at least 20 petechiae 65. These vaccines are given 3 doses at one month within 1cm square intervals: C. Steadily increasing hematocrit count A. DPT, BCG, TT D. Fall in the platelet count B. OPV, HEP. B, DPT C. DPT, TT, OPV 60. Which of the following is the most important D. Measles, OPV, DPT treatment of patients with Dengue H-fever? A. Give aspirin for fever Situation – With the increasing documented cases of B. Replacement of body fluids CANCER the best alternative to treatment still remains to C. Avoid unnecessary movement of patient be PREVENTION. The following conditions apply. D. Ice cap over the abdomen in case of melena 66. Which among the following is the primary focus of prevention of cancer? Situation: Health education and Health promotion is an A. Elimination of conditions causing cancer important part of nursing responsibility in the B. Diagnosis and treatment community. Immunization is a form of health promotion C. Treatment at early stage that aims at preventing the common childhood illnesses. D. Early detection 61. In correcting misconceptions and myths about 67. In the prevention and control of cancer, which of certain diseases and their management, the the following activities is the most important